Как решать подкоренные выражения: Подкоренные выражения: как решать

n=a$.

Содержание

Свойства корня с примерами

Основное свойство корня: величина корня не изменится, если показатель корня и показатель подкоренного выражения умножить или разделить на одно и то же число. Отсюда следует, что корни разных степеней можно привести к одинаковым показателям.

Приведём простой пример.

Пример 2

Рисунок 1. Пример. Автор24 — интернет-биржа студенческих работ

Также имеются следующее свойства:

Рисунок 2. Свойства. Автор24 — интернет-биржа студенческих работ

Рисунок 3. Свойства. Автор24 — интернет-биржа студенческих работ

Рисунок 4. Свойства. Автор24 — интернет-биржа студенческих работ

Решим простые примеры.

Пример 3

Рисунок 5. Пример. Автор24 — интернет-биржа студенческих работ

Пример 4

Рисунок 6. Пример. Автор24 — интернет-биржа студенческих работ

Преобразования корней

При решении корней кроме вышеуказанных свойств применяют простейшие преобразования, такие как

  • вынесение множителей за знак корня

Рисунок 7. Вынесение множителей за знак корня. Автор24 — интернет-биржа студенческих работ

  • подведение множителей под знак корня

Рисунок 8. Подведение множителей под знак корня. Автор24 — интернет-биржа студенческих работ

  • освобождение подкоренного выражения от знаменателей.

Для последнего случая приведём пример с числами:

Рисунок 9. Освобождение подкоренного выражения от знаменателей. Автор24 — интернет-биржа студенческих работ

Рассмотрим пример, в котором показывается как происходит освобождение знаменателя дроби от корней.

Пример 5

Имеем:

Рисунок 10. Пример. Автор24 — интернет-биржа студенческих работ

Произведём умножение членов дробей на $\sqrt5$:

Рисунок 11. Пример. Автор24 — интернет-биржа студенческих работ

Пример решения

На практике необходимо помнить все перечисленные свойства и преобразования корней, а также свойства степеней, которые, как мы условились, читателю уже известны. 2}=9+7=16.$

Иррациональные выражения (выражения с корнями) и их преобразование

Статья раскрывает смысл иррациональных выражений  и преобразования с ними. Рассмотрим само понятие иррациональных выражений, преобразование и характерные выражения.

Что такое иррациональные выражения?

При знакомстве с корнем  в школе мы изучаем понятие иррациональных выражений. Такие выражения тесно связаны с корнями.

Определение 1

Иррациональные выражения – это выражения, которые имеют корень. То есть это выражения, имеющие радикалы.

Основываясь на данном определении, мы имеем, что x-1, 83·36-12·3, 7-4·3·(2+3), 4·a2d5:d92·a35 — это все выражения иррационального типа.

При рассмотрении выражения x·x-7·x+7x+32·x-83 получаем, что выражение является рациональным. К рациональным выражениям  относят многочлены и алгебраические дроби. Иррациональные включают в себя работу с логарифмическими выражениями или подкоренными выражениями.

Основные виды преобразований иррациональных выражений

При вычислении таких выражений необходимо обратить внимание на ОДЗ.  Часто они требуют дополнительных преобразований в виде раскрытия скобок, приведения подобных членов, группировок и так далее. Основа таких преобразований – действия с числами. Преобразования иррациональных выражений придерживаются строгого порядка.

Пример 1

Преобразовать выражение 9+33-2+4·33+1-2·33.

Решение

Необходимо выполнить замену числа 9 на выражение, содержащее корень. Тогда получаем, что

81+33-2+4·33+1-2·33==9+33-2+4·33+1-2·33

Полученное выражение имеет подобные слагаемые, поэтому выполним приведение и группировку. Получим

9+33-2+4·33+1-2·33==9-2+1+33+4·33-2·33==8+3·33
Ответ: 9+33-2+4·33+1-2·33=8+3·33

Пример 2

Представить выражение x+352-2·x+35+1-9 в виде произведения двух иррациональных с использованием формул сокращенного умножения.

Решения

x+352-2·x+35+1-9==x+35-12-9

Представляем 9 в виде 32, причем применим формулу разности квадратов:

x+35-12-9=x+35-12-32==x+35-1-3·x+35-1+3==x+35-4·x+35+2

Результат тождественных преобразований привел к произведению двух рациональных выражений, которые необходимо было найти.

Ответ: 

x+352-2·x+35+1-9==x+35-4·x+35+2

Можно выполнять ряд других преобразований, которые относятся к иррациональным выражениям.

Преобразование подкоренного выражения

Важно то, что выражение, находящееся под знаком корня, можно заменить на тождественно равное ему. Данное утверждение дает возможность работать с подкоренным выражением. К примеру, 1+6 можно заменить на 7 или 2·a54-6 на 2·a4·a4-6. Они тождественно равные, поэтому замена имеет смысл.

Когда не существует а1, отличное от a, где справедливо неравенство вида an=a1n, тогда такое равенство возможно только при а=а1. Значения таких выражений равны с любыми значениями переменных.

Использование свойств корней

Свойства корней применяют для упрощения выражений. Чтобы применить свойство a·b=a·b, где a≥0, b≥0, тогда из иррационального  вида 1+3·12 можно стать тождественно равным 1+3·12. Свойство …ankn2n1=an1·n2·,…,·nk , где a≥0 говорит о том, что x2+443 можно записать в форме x2+424.

Имеются некоторые нюансы при преобразовании подкоренных выражений. Если имеется выражение, то -7-814=-74-814 записать не можем, так как формула abn=anbn служит только для неотрицательного a и положительного b. Если свойство применить правильно, тогда получится выражение вида 74814.

Для правильного преобразования используют преобразования иррациональных выражений с использованием свойств корней.

Внесение множителя под знак корня

Определение 3

Внести под знак корня – значит заменить выражение B·Cn, а B и C являются некоторыми числами или выражениями, где n – натуральное число, которое больше 1, равным выражением, которое имеет вид Bn·Cn или -Bn·Cn.

Нужна помощь преподавателя?

Опиши задание — и наши эксперты тебе помогут!

Описать задание

Если упростить выражение вида 2·x3, то после внесения под корень, получаем, что 23·x3. Такие преобразования возможны только после подробного изучения правил внесения множителя под знак корня.

Вынесение множителя из-под знака корня

Если имеется выражение вида Bn·Cn, тогда его приводят к виду B·Cn, где имеется нечетные n, которые принимают вид B·Cn с четными n, В и C являются некоторыми числами и выражениями.

То есть, если брать иррациональное выражение вида 23·x3, вынести множитель из-под корня, тогда получим выражение 2·x3. Или x+12·7 даст в результате выражение вида x+1·7, которое имеет еще одну запись в виде x+1·7.

Вынесение множителя из-под корня необходимо для упрощения выражения и его быстрого преобразования.

Преобразование дробей, содержащих корни

Иррациональное выражение может быть как натуральным числом, так и в виде дроби. Для преобразования дробных выражений большое внимание обращают на его знаменатель. Если взять дробь вида (2+3)·x4x2+53, то числитель примет вид 5·x4, а, использовав свойства корней, получим, что знаменатель станет x2+56. Исходную дробь можно будет записать в виде 5·x4x2+56.

Необходимо обратить внимание на то, что необходимо изменять знак только числителя или только знаменателя. Получим, что

-x+2·x-3·x2+74=x+2·x-(-3·x2+74)=x+2·x3·x2-74

Сокращение дроби чаще всего используется при упрощении. Получаем, что

3·x+43-1·xx+43-13 сокращаем на x+43-1. Получим выражение 3·xx+43-12.

Перед сокращением необходимо выполнять преобразования, которые упрощают выражение и дают возможность разложить на множители сложное выражение. Чаще всего применяют формулы сокращенного умножения.

Если взять дробь вида 2·x-yx+y, то необходимо вводить новые переменные u=x и v=x, тогда заданное выражение поменяет вид и станет 2·u2-v2u+v. Числитель следует разложить на многочлены по формуле, тогда получим, что

2·u2-v2u+v=2·(u-v)·u+vu+v=2·u-v. После выполнения обратной замены придем к виду 2·x-y, которое равно исходному.

Допускается приведение к новому знаменателю, тогда необходимо числитель умножать на дополнительный множитель. Если взять дробь вида x3-10,5·x, тогда приведем к знаменателю x. для этого  нужно умножить числитель и знаменатель на выражение 2·x, тогда получаем выражение x3-10,5·x=2·x·x3-10,5·x·2·x=2·x·x3-1x.

Сокращение дробей или приведение подобных необходимо только на ОДЗ указанной дроби. При умножении числителя и знаменателя на иррациональное выражение получаем, что  мы избавляемся от иррациональности в знаменателе.

Избавление от иррациональности в знаменателе

Когда выражение избавляется от корня в знаменателе путем преобразования, то это называется избавлением от иррациональности. Рассмотрим на примере дроби вида x33. После избавления от иррациональности получаем новую дробь вида 93·x3.

Переход от корней к степеням

Переходы от корней к степеням необходимы для быстрого преобразования иррациональных выражений. Если рассмотреть равенство amn=amn, то видно, что его использование возможно, когда a является положительным числом, m –целым числом, а n – натуральным. Если рассматривать выражение 5-23, то иначе имеем право записать его как 5-23. Эти выражения равнозначны.

Когда под корнем имеется отрицательное число или число с переменными, тогда формула amn=amn не всегда применима. Если нужно заменить такие корни (-8)35 и (-16)24 степенями, тогда получаем, что -835 и -1624 по формуле amn=amn не работаем с отрицательными а. для того, чтобы подробно разобрать тему подкоренных выражений и их упрощений, необходимо изучать статью о переходе от корней к степеням и обратно. Следует помнить о том, что формула amn=amn применима не для всех выражений такого вида. Избавление от иррациональности способствует дальнейшему упрощению выражения, его преобразованию и решению.

Действия со степенями и корнями

1. При умножении степеней с одинаковыми основаниями показатели
складываются, а основание остаётся прежним:

.

Например, .

2. При делении степеней с одинаковыми основаниями показатели
степеней вычитаются, а основание остаётся прежним:

.

Например, .

3. При возведении степени в степень показатели
степеней перемножаются, а основание остаётся прежним:

.

Например, .

4. Степень произведения равна произведению степеней множителей:

.

Например, .

5. Степень частного равна частному степеней делимого и делителя:

.

Например, .

Пример 1. Найти значение выражения

.

Решение. В данном случае в явной форме ни одно из свойств степени
с натуральным показателем применить нельзя, так как все степени имеют разные основания.
Запишем некоторые степени в другом виде:


(степень произведения равна произведению степеней множителей),


(при умножении степеней с одинаковыми основаниями показатели складываются, а основание
остаётся прежним, при возведении степени в степень показатели степеней перемножаются,
а основание остаётся прежним).

Теперь получим:

В данном примере были использованы первые четыре свойства степени
с натуральным показателем.

Свойства степеней и корней интенсивно используются при упрощении выражений в задачах
математического анализа, например, для нахождения производной параметрически заданной функции и
производной функции, заданной неявно.

Имеют место следующие тождества:

1) ;

2) ;

3) .

Выполнить действия со степенями самостоятельно, а затем посмотреть решения

Пример 2. Найти значение выражения

.

Пример 3. Найти значение выражения

.

1. Корень k-й степени из произведения неотрицательных
чисел равен произведению корней той же степени из сомножителей:
,
где
(правило извлечения корня из произведения).

2. Если ,
то
(правило извлечения корня из дроби).

3. Если ,
то
(правило извлечения корня из корня).

4. Если ,
то
(правило возведения корня в степень).

5. Если ,
то ,
где ,
т. е. показатель корня и показатель подкоренного выражения можно умножить на одно и
то же число.

6. Если ,
то ,
т. е. большему положительному подкоренному выражению соответствует и большее значение
корня.

7. Все указанные выше формулы часто применяются в обратном порядке
(т. е. справа налево). Например:


(правило умножения корней),


(правило деления корней),

.

8. Правило вынесения множителя из-под знака корня. При
.

9. Обратная задача — внесение множителя под знак корня. Например,

10. Уничтожение иррациональности в знаменателе дроби. Рассмотрим
некоторые типичные случаи.

а) ,
так как .

Например, .

б)

Например,

в)

и т. д.

Другие темы в блоке «Школьная математика»

Иррациональные выражения. Преобразование иррациональных выражений

Выражения, содержащие корень, который нельзя извлечь, называются иррациональными или радикальными.

Примеры:

  — иррациональные выражения.

Сложение и вычитание корней

При сложении или вычитании иррациональных выражений их пишут одно за другим с сохранением их знаков.

Примеры:

В некоторых случаях с помощью преобразования можно сделать иррациональные выражения подобными, то есть, имеющими одинаковые показатели корней и подкоренные числа (или выражения), а затем сделать приведение.

Примеры:

Умножение и деление корней

При умножении иррациональных выражений с одинаковыми показателями корней перемножаются их подкоренные числа или выражения:

При делении иррациональных выражений с одинаковыми показателями корней подкоренное число или выражение делимого делится на подкоренное число или выражение делителя:

Примеры:

Возведение корня в степень

Чтобы возвести в степень иррациональное выражение, следует возвести в степень подкоренное число или выражение:

Примеры:

При возведении    в  n-ю  степень знак корня отбрасывается, так как возведение числа (или выражения) в  n-ю  степень и извлечение из него корня  n-ой  степени — это взаимно сокращающиеся действия:

Извлечение корня

Чтобы извлечь корень из иррационального выражения, следует показатели корней перемножить:

,  так как 

Пример.

С помощью таких преобразований можно упростить извлечение корней  4-й,  6-й,  8-й,  9-й  и т. п. степеней из чисел.

Примеры:

Сокращение корней

Величина иррационального выражения не изменится, если показатель корня и подкоренного выражения умножить или разделить на одно и то же число:

так как извлечение корня и возведение в степень — это взаимно сокращающиеся действия, если их показатели равны.

На этом свойстве основано сокращение корней и приведение их к общему показателю.

Сокращение корней — это деление показателей корня и подкоренного числа (или выражения) на одно и то же число, если оно является общим множителем для всех показателей.

Примеры:

Приведение корней к общему показателю

Приведение корней к общему показателю имеет большое сходство с приведением дробей к общему знаменателю. Рассмотрим два способа:

  1. Показатели корней не имеют общих множителей. В этом случае показатель каждого корня и его подкоренное число (или выражение) умножают на произведение остальных корней.

    Рассмотрим три выражения:

    ,

    Так как у данных показателей нет общего множителя, то просто перемножаем все показатели между собой. Полученный результат и станет общим показателем. После приведения к общему показателю выражения будут иметь следующий вид:

  2. Показатели корней имеют общий множитель. В этом случае надо найти НОК показателей и умножить показатель каждого корня на недостающий множитель.

    Рассмотрим два выражения:

    ,

    НОК (4, 6) = 12,  значит, для первого выражения дополнительным множителем будет  3,  а для второго  2.  После приведения к общему показателю выражения будут иметь следующий вид:

При умножении и делении иррациональных выражений с разными показателями их приводят к общему показателю, а затем уже умножают или делят их подкоренные числа или выражения.

Примеры:

Вычислить значение выражения с корнями

Умение работать с числовыми выражениями, содержащими квадратный корень, необходимо для успешного решения ряда задач из ОГЭ и ЕГЭ. Как правило, на этих экзаменах достаточно базового представления о том, что такое извлечение корня и как оно осуществляется на практике.

Определение

Корень степени n из числа X – это такое число x, для которого верно равенство: x n = X.

Найти значение выражения с корнем – это значит найти x при известных X и n.

Квадратный корень или, что то же самое, корень второй степени из X – число x, для которого выполнено равенство: x 2 = X.

Обозначение: ∛Х. Здесь 3 – степень корня, Х – подкоренное выражение. Знак ‘√’ часто называют радикалом.

Если над корнем не стоит число, указывающее на степень, то по умолчанию подразумевается степень 2.

В школьном курсе для четных степеней обычно не рассматривают отрицательные корни и подкоренные выражения. Например, не существует √-2, а для выражения √4 верным ответом считается 2, несмотря на то, что (-2) 2 тоже равняется 4.

Рациональность и иррациональность корней

Наиболее простое из возможных заданий c корнем – найти значение выражения либо проверить его на рациональность.

Например, вычислить значения √25; ∛8; ∛-125:

  • √25 = 5, так как 5 2 = 25;
  • ∛8 = 2, так как 2 3 = 8;
  • ∛ – 125 = -5, так как (-5) 3 = -125.

Ответы в приведенных примерах – это рациональные числа.

При работе с выражениями, не содержащими буквенных констант и переменных, рекомендуется всегда выполнять подобную проверку с помощью обратной операции возведения в натуральную степень. Нахождение числа x в n-й степени эквивалентно вычислению произведения n множителей x.

Существует множество выражений с корнем, значение которых иррационально, то есть записывается в виде бесконечной непериодической дроби.

По определению рациональные – это те, что можно выразить обыкновенной дробью, а иррациональные – все остальные действительные числа.

К таким относятся √24, √0,1, √101.

Если в задачнике сказано: найдите значение выражения с корнем из 2, 3, 5, 6, 7 и т. д., то есть из тех натуральных чисел, которые не содержатся в таблице квадратов, то в правильном ответе √2 может присутствовать (когда не оговорено обратное).

Проведение оценки

В задачах с открытым ответом, если найти значение выражения с корнем и записать его рациональным числом невозможно, результат следует оставить в виде радикала.

Некоторые задания могут потребовать проведения оценки. Например, сравнить 6 и √37. Для решения требуется возвести оба числа в квадрат и сравнить результаты. Из двух чисел больше то, чей квадрат больше. Данное правило работает для всех положительных чисел:

Точно так же решаются задачи, в которых несколько чисел надо расставить в порядке возрастания или убывания.

Пример: расставить по возрастанию 5, √6, √48, √√64.

После возведения в квадрат имеем: 25, 6, 48, √64. 2 = 4 cdot 7 = 28) .

Найдите значение выражения (frac <sqrt<12>cdot sqrt<540>><sqrt<30>>) .

Перепишем исходное выражение, занеся все числа под один корень:

Разложим выражение под корнем на множители так, чтобы среди них были полные квадраты:

(sqrt <216>= sqrt <4 cdot 9 cdot 6>= 2 cdot 3 cdot sqrt <6>= 6sqrt<6>) .

Найдите значение выражения (frac <sqrt<150>cdot sqrt<216>><sqrt<90>>) .

Перепишем исходное выражение, занеся все числа под один корень:

Разложим выражение под корнем на множители так, чтобы среди них были полные квадраты:

Найдите значение выражения (4sqrt <3>cdot sqrt <2>cdot 4sqrt<6>) .

Преобразуем (sqrt <6>= sqrt <2>cdot sqrt<3>) .

Найдем произведение множителей без корня, а множители с корнем сгруппируем:

(4sqrt <3>cdot sqrt <2>cdot 4 sqrt <2>cdot sqrt <3>=16 cdot sqrt<3>^2 cdot sqrt<2>^2 = 16 cdot 3 cdot 2 = 96) .

Этот видеоурок доступен по абонементу

У вас уже есть абонемент? Войти

Повторение свойств квадратных корней

Вкратце повторим теорию и напомним основные свойства квадратных корней.

Свойства квадратных корней:

1. 2. ;

3. ;

4. .

Примеры на упрощение выражений с корнями

Перейдем к примерам использования этих свойств.

Пример 1. Упростить выражение .

Решение. Для упрощения число 120 необходимо разложить на простые множители:

. Квадрат суммы раскроем по соответствующей формуле:

.

Пример 2. Упростить выражение .

Решение. Учтем, что данное выражение имеет смысл не при всех возможных значениях переменной, т. к. в данном выражении присутствуют квадратные корни и дроби, что приводит к «сужению» области допустимых значений. ОДЗ: Ответ. Пример 3. Упростить выражение .

Решение. Видно, что вторая скобка числителя имеет неудобный вид и нуждается в упрощении, попробуем разложить ее на множители с помощью метода группировки.

. Для возможности выносить общий множитель мы упростили корни путем их разложения на множители. Подставим полученное выражение в исходную дробь:

. После сокращения дроби применяем формулу разности квадратов.

Пример на избавление от иррациональности

Пример 4. Освободиться от иррациональности (корней) в знаменателе: а)

Читайте также:  Вход в личную почту на яндексе

.

.

Ответ.Пример на доказательство и на выделение полного квадрата в сложном радикале

Пример 5. Докажите равенство .

Доказательство. Воспользуемся определением квадратного корня, из которого следует, что квадрат правого выражения должен быть равен подкоренному выражению:

, получили верное равенство.

Пример 6. Упростить выражение .

Решение. Указанное выражение принято называть сложным радикалом (корень под корнем). В данном примере необходимо догадаться выделить полный квадрат из подкоренного выражения. Для этого заметим, что из двух слагаемых . Подставим это выражение под корень:

Ответ..

На этом занятии мы заканчиваем тему «Функция . Свойства квадратного корня», а на следующем уроке начинаем новую тему «Действительные числа».

1. Башмаков М.И. Алгебра 8 класс. – М.: Просвещение, 2004.

2. Дорофеев Г.В., Суворова С.Б., Бунимович Е.А. и др. Алгебра 8. – 5-е изд. – М.: Просвещение, 2010.

3. Никольский С.М., Потапов М.А., Решетников Н.Н., Шевкин А.В. Алгебра 8 класс. Учебник для общеобразовательных учреждений. – М.: Просвещение, 2006.

Дополнительные рекомендованные ссылки на ресурсы сети Интернет

1. Интернет-портал xenoid.ru (Источник).

2. Математическая школа (Источник).

3. Интернет-портал XReferat.Ru (Источник).

1. №357, 360, 372, 373, 382. Дорофеев Г.В., Суворова С.Б., Бунимович Е.А. и др. Алгебра 8. – 5-е изд. – М.: Просвещение, 2010.

2. Избавьтесь от иррациональности в знаменателе: а) 3. Упростите выражение: а) 4. Докажите тождество .

Если вы нашли ошибку или неработающую ссылку, пожалуйста, сообщите нам – сделайте свой вклад в развитие проекта.

Преобразования иррациональных выражений | О математике понятно

Иррациональные выражения и их преобразования

        В прошлый раз мы вспомнили (или узнали — кому как), что же такое корень n-й степени, научились извлекать такие корни, разобрали по винтикам основные свойства корней и решали несложные примеры с корнями.

        Этот урок будет продолжением предыдущего и будет посвящён преобразованиям самых разных выражений, содержащих всевозможные корни. Такие выражения называются иррациональными. Здесь появятся и выражения с буквами, и дополнительные условия, и избавление от иррациональности в дробях, и некоторые продвинутые приёмы в работе с корнями. Те приёмы, которые будут рассматриваться в данном уроке, станут хорошей базой для решения задач ЕГЭ (и не только) практически любого уровня сложности. Итак, давайте приступим.

        Прежде всего я продублирую здесь основные формулы и свойства корней. Чтобы не скакать из темы в тему. Вот они:

 при 

        Формулы эти надо обязательно знать и уметь применять. Причём в обе стороны — как слева направо, так и справа налево. Именно на них и основывается решение большинства заданий с корнями любой степени сложности. Начнём пока с самого простого — с прямого применения формул или их комбинаций.

Простое применение формул

        В этой части будут рассматриваться простые и безобидные примеры — без букв, дополнительных условий и прочих хитростей. Однако даже в них, как правило, имеются варианты. И чем навороченнее пример, тем больше таких вариантов. И у неопытного ученика возникает главная проблема — с чего начинать? Ответ здесь простой — не знаешь, что нужно — делай что можно. Лишь бы ваши действия шли в мире и согласии с правилами математики и не противоречили им.) Например, такое задание:

        Вычислить: 

        Даже в таком простеньком примере возможны несколько путей к ответу.

        Первый — просто перемножить корни по первому свойству и извлечь корень из результата:

        Второй вариант такой:  не трогаем, работаем с . Выносим множитель из-под знака корня, а дальше — по первому свойству. Вот так:

        Решать можно как больше нравится. В любом из вариантов ответ получается один — восьмёрка. Мне, например, проще перемножить 4 и 128 и получить 512, а из этого числа отлично извлекается кубический корень. Если кто-то не помнит, что 512 — это 8 в кубе, то не беда: можно записать 512 как 29 (первые 10 степеней двойки, я надеюсь, помните?) и по формуле корня из степени:

        Другой пример.

        Вычислить: .

        Если работать по первому свойству (всё загнать под один корень), то получится здоровенное число, из которого корень потом извлекать — тоже не сахар. Да и не факт, что он извлечётся ровно.) Поэтому здесь полезно в числе  вынести множители из-под корня. Причём вынести по максимуму:

        И теперь всё наладилось:

        Осталось восьмёрку и двойку записать под одним корнем (по первому свойству) и — готово дело. 🙂

        Добавим теперь немного дробей.

        Вычислить:

        

        Пример совсем примитивный, однако и в нём имеются варианты. Можно с помощью вынесения множителя преобразовать числитель и сократить со знаменателем:

        А можно сразу воспользоваться формулой деления корней:

        Как видим, и так, и сяк — всяко правильно.) Если не споткнуться на полпути и не ошибиться. Хотя где тут ошибаться-то…

        Разберём теперь самый последний пример из домашнего задания прошлого урока:

        Упростить:

            

        Совершенно немыслимый набор корней, да ещё и вложенных. Как быть? Главное — не бояться! Здесь мы первым делом замечаем под корнями числа 2, 4 и 32 — степени двойки. Первое что нужно сделать — привести все числа к двойкам: всё-таки чем больше одинаковых чисел в примере и меньше разных, тем проще.) Начнём отдельно с первого множителя:

        Число  можно упростить, сократив двойку под корнем с четвёркой в показателе корня:

        Теперь, согласно корню из произведения:

.

        В числе  выносим двойку за знак корня: 

        А с выражением  расправляемся по формуле корня из корня:

        Значит, первый множитель запишется вот так:

.

        Вложенные корни исчезли, числа стали поменьше, что уже радует. Вот только корни разные, но пока так и оставим. Надо будет — преобразуем к одинаковым. Берёмся за второй множитель.)

        Второй множитель преобразовываем аналогично, по формуле корня из произведения и корня из корня. Где надо — сокращаем показатели по пятой формуле:

        Вставляем всё в исходный пример и получаем:

        Получили произведение целой кучи совершенно разных корней. Неплохо было бы привести их все к одному показателю, а там — видно будет. Что ж, это вполне возможно. Наибольший из показателей корней равен 12, а все остальные — 2, 3, 4, 6 — делители числа 12. Поэтому будем приводить все корни по пятому свойству к одному показателю — к 12:

        Считаем и получаем:

        Красивого числа не получили, ну и ладно. Нас просили упростить выражение, а не посчитать. Упростили? Конечно! А вид ответа (целое число или нет) здесь уже не играет никакой роли.

Немного сложения / вычитания и формул сокращённого умножения

        К сожалению, общих формул для сложения и вычитания корней в математике нету. Однако, в заданиях сплошь и рядом встречаются эти действия с корнями. Здесь необходимо понимать, что любые корни — это точно такие же математические значки, как и буквы в алгебре.) И к корням применимы те же самые приёмы и правила, что и к буквам — раскрытие скобок, приведение подобных, формулы сокращённого умножения и т.п.

        Например, каждому ясно, что . Точно так же одинаковые корни можно совершенно спокойно между собой складывать/вычитать:

        Если корни разные, то ищем способ сделать их одинаковыми — внесением/вынесением множителя или же по пятому свойству. Если ну никак не упрощается, то, возможно, преобразования более хитрые.

        Смотрим первый пример.

        Найти значение выражения: .

        Все три корня хоть и кубические, но из разных чисел. Чисто не извлекаются и между собой складываются/вычитаются. Стало быть, применение общих формул здесь не катит. Как быть? А вынесем-ка множители в каждом корне. Хуже в любом случае не будет.) Тем более что других вариантов, собственно, и нету:

        Стало быть, .

        Вот и всё решение. Здесь мы от разных корней перешли к одинаковым с помощью вынесения множителя из-под корня. А затем просто привели подобные.) Решаем дальше.

        Найти значение выражения

        С корнем из семнадцати точно ничего не поделаешь. Работаем по первому свойству — делаем из произведения двух корней один корень:

        А теперь присмотримся повнимательнее. Что у нас под большим кубическим корнем? Разность ква.. Ну, конечно! Разность квадратов:

        Теперь осталось только извлечь корень: .

        Дальше очень похожий пример, но посложнее.

        Вычислить: 

        Здесь придётся проявить математическую смекалку.) Мыслим примерно следующим образом: «Так, в примере произведение корней. Под одним корнем разность, а под другим — сумма. Очень похоже на формулу разности квадратов. Но… Корни — разные! Первый квадратный, а второй — четвёртой степени… Хорошо бы сделать их одинаковыми. По пятому свойству можно легко из квадратного корня сделать корень четвёртой степени. Для этого достаточно подкоренное выражение возвести в квадрат.»

        Если вы мыслили примерно так же, то вы — на полпути к успеху. Совершенно верно! Превратим первый множитель в корень четвёртой степени. Вот так:

       Теперь, ничего не поделать, но придётся вспомнить формулу квадрата разности. Только в применении к корням. Ну и что? Чем корни хуже других чисел или выражений?! Возводим:

       «Хм, ну возвели и что? Хрен редьки не слаще. Стоп! А если вынести четвёрку под корнем? Тогда выплывет то же самое выражение, что и под вторым корнем, только с минусом, а ведь именно этого мы и добиваемся!»

        Верно! Выносим четвёрку:

.

        А теперь — дело техники:

.

        Вот так распутываются сложные примеры. ) Теперь пора потренироваться с дробями.

        Вычислить:

        

        Ясно, что надо преобразовывать числитель. Как? По формуле квадрата суммы, разумеется. У нас есть ещё варианты разве? 🙂 Возводим в квадрат, выносим множители, сокращаем показатели (где надо):

        Во как! Получили в точности знаменатель нашей дроби. ) Значит, вся дробь, очевидно, равна единице:

        Ещё пример. Только теперь на другую формулу сокращённого умножения.)

        Вычислить:

            

        Понятно, что квадрат разности надо в дело применять. Выписываем знаменатель отдельно и — поехали!

        Выносим множители из-под корней:

        Следовательно, 

.

        Теперь всё нехорошее великолепно сокращается и получается:

        Что ж, поднимаемся на следующий уровень. 🙂

Буквы и дополнительные условия

        Буквенные выражения с корнями — штука более хитрая, чем числовые выражения, и является неиссякаемым источником досадных и очень грубых ошибок. Перекроем этот источник.) Ошибки всплывают из-за того, что частенько таких заданиях фигурируют отрицательные числа и выражения. Они либо даны нам прямо в задании, либо спрятаны в буквах и дополнительных условиях. А нам в процессе работы с корнями постоянно надо помнить, что в корнях чётной степени как под самим корнем, так и в результате извлечения корня должно быть неотрицательное выражение. Ключевой формулой в задачах этого пункта будет четвёртая формула:

        С корнями нечётной степени вопросов никаких — там всегда всё извлекается что с плюсом, что с минусом. И минус, если что, выносится вперёд. Будем сразу разбираться с корнями чётных степеней.) Например, такое коротенькое задание.

        Упростить: , если .

        Казалось бы, всё просто. Получится просто икс. ) Но зачем же тогда дополнительное условие  ? В таких случаях полезно прикинуть на числах. Чисто для себя.) Если , то икс — заведомо отрицательное число. Минус три, например. Или минус сорок. Пусть . Можно минус три возвести в четвёртую степень? Конечно! Получится 81. Можно из 81 извлечь корень четвёртой степени? А почему нет? Можно! Получится тройка. Теперь проанализируем всю нашу цепочку:

        Что мы видим? На входе было отрицательное число, а на выходе — уже положительное. Было минус три, стало плюс три.) Возвращаемся к буквам. Вне всяких сомнений, по модулю это будет точно икс, но только сам икс у нас с минусом (по условию!), а результат извлечения (в силу арифметического корня!) должен быть с плюсом. Как получить плюс? Очень просто! Для этого достаточно перед заведомо отрицательным числом поставить минус.) И правильное решение выглядит так:

        Кстати сказать, если бы мы воспользовались формулой , то, вспомнив определение модуля, сразу получили бы верный ответ. Поскольку

|x| = -x при x<0.

        Дальше тренируемся.)

        Вынести множитель за знак корня: , где .

        Первый взгляд — на подкоренное выражение. Тут всё ОК. При любом раскладе оно будет неотрицательным. Начинаем извлекать. По формуле корня из произведения, извлекаем корень из каждого множителя:

        Откуда взялись модули, объяснять, думаю, уже не надо.) А теперь анализируем каждый из модулей.

        Множитель |a| так и оставляем без изменений: у нас нету никакого условия на букву a. Мы не знаем, положительное она или отрицательная. Следующий модуль |b2| можно смело опустить: в любом случае выражение b2 неотрицательно. А вот насчёт |c3| — тут уже задачка.) Если , то и c3<0. Стало быть, модуль надо раскрыть с минусом: |c3| = —c3. Итого верное решение будет такое:

        А теперь — обратная задача. Не самая простая, сразу предупреждаю!

        Внести множитель под знак корня: .

        Если вы сразу запишете решение вот так

,

то вы попали в ловушку. Это неверное решение! В чём же дело?

        Давайте вглядимся в выражение под корнем . Под корнем четвёртой степени, как мы знаем, должно находиться неотрицательное выражение. Иначе корень смысла не имеет.) Поэтому  А это, в свою очередь, значит, что  и, следовательно, само  также неположительно: .

        И ошибка здесь состоит в том, что мы вносим под корень неположительное число : четвёртая степень превращает его в неотрицательное и получается неверный результат — слева заведомый минус, а справа уже плюс. А вносить под корень чётной степени мы имеем право только неотрицательные числа или выражения. А минус, если есть, оставлять перед корнем.) Как же нам выделить неотрицательный множитель в числе , зная, что оно само стопудово отрицательное? Да точно так же! Поставить минус.) А чтобы ничего не поменялось, скомпенсировать его ещё одним минусом. Вот так:

        И теперь уже неотрицательное число (-b) спокойно вносим под корень по всем правилам:

        Этот пример наглядно показывает, что, в отличие от других разделов математики, в корнях правильный ответ далеко не всегда вытекает автоматически из формул. Необходимо подумать и лично принять верное решение.) Особенно следует быть внимательнее со знаками в иррациональных уравнениях и неравенствах.

        Разбираемся со следующим важным приёмом в работе с корнями — избавлением  от иррациональности.

Избавление от иррациональности в дробях

        Если в выражении присутствуют корни, то, напомню, такое выражение называется выражением с иррациональностью. В некоторых случаях бывает полезно от этой самой иррациональности (т.е. корней) избавиться. Как можно ликвидировать корень? Корень у нас пропадает при… возведении в степень. С показателем либо равным показателю корня, либо кратным ему. Но, если мы возведём корень в степень (т.е. помножим корень сам на себя нужное число раз), то выражение от этого поменяется. Нехорошо.) Однако в математике бывают темы, где умножение вполне себе безболезненно. В дробях, к примеру. Согласно основному свойству дроби, если числитель и знаменатель умножить (разделить) на одно и то же число, то значение дроби не изменится.

        Допустим, нам дана вот такая дробь:

        Можно ли избавиться от корня в знаменателе? Можно! Для этого корень надо возвести в куб. Чего нам не хватает в знаменателе для полного куба? Нам не хватает множителя , т.е. . Вот и домножаем числитель и знаменатель дроби на 

        Корень в знаменателе исчез. Но… он появился в числителе. Ничего не поделать, такова судьба.) Нам это уже не важно: нас просили знаменатель от корней освободить. Освободили? Безусловно.)

        Кстати, те, кто уже в ладах с тригонометрией, возможно, обращали внимание на то, что в некоторых учебниках и таблицах, к примеру,  обозначают по-разному: где-то , а где-то . Вопрос — что правильно? Ответ: всё правильно! ) Если догадаться, что  – это просто результат освобождения от иррациональности в знаменателе дроби . 🙂

        Зачем нам освобождаться от иррациональности в дробях? Какая разница — в числителе корень сидит или в знаменателе? Калькулятор всё равно всё посчитает.) Ну, для тех, кто не расстаётся с калькулятором, разницы действительно практически никакой… Но, даже считая на калькуляторе, можно обратить внимание на то, что делить на целое число всегда удобнее и быстрее, чем на иррациональное. А уж про деление в столбик вообще умолчу.)

        Следующий пример только подтвердит мои слова.

        Освободиться от иррациональности в знаменателе дроби:

        Как здесь ликвидировать квадратный корень в знаменателе? Если числитель и знаменатель помножить на выражение , то в знаменателе получится квадрат суммы. Сумма квадратов первого и второго чисел дадут нам просто числа безо всяких корней, что очень радует. Однако… всплывёт удвоенное произведение первого числа на второе, где корень из трёх всё равно останется. Не канает. Как быть? Вспомнить другую замечательную формулу сокращённого умножения! Где никаких удвоенных произведений, а только квадраты:

        Такое выражение, которое при домножении какой-то суммы (или разности) выводит на разность квадратов, ещё называют сопряжённым выражением. В нашем примере сопряжённым выражением будет служить разность . Вот и домножаем на эту разность числитель и знаменатель:

        Что тут можно сказать? В результате наших манипуляций не то что корень из знаменателя исчез — вообще дробь исчезла! 🙂 Даже с калькулятором отнять корень из трёх от тройки проще, чем считать дробь с корнем в знаменателе. Ещё пример.

        Освободиться от иррациональности в знаменателе дроби:

        Как здесь выкручиваться? Формулы сокращённого умножения с квадратами сразу не катят — не получится полной ликвидации корней из-за того, что корень у нас в этот раз не квадратный, а кубический. Надо, чтобы корень как-то возвёлся в куб. Стало быть, применять надо какую-то из формул с кубами. Какую? Давайте подумаем. В знаменателе — сумма . Как нам добиться возведения корня в куб? Домножить на неполный квадрат разности! Значит, применять будем формулу суммы кубов. Вот эту:

        В качестве a у нас тройка, а в качестве b — корень кубический из пяти:

        И снова дробь исчезла.) Такие ситуации, когда при освобождении от иррациональности в знаменателе дроби у нас вместе с корнями полностью исчезает сама дробь, встречаются очень часто. Как вам вот такой примерчик!

        Вычислить:

        

        Попробуйте просто сложить эти три дроби! Без ошибок! 🙂 Один общий знаменатель чего стоит. А что, если попробовать освободиться от иррациональности в знаменателе каждой дроби? Что ж, пробуем:

        Ух ты, как интересно! Все дроби пропали! Напрочь. И теперь пример решается в два счёта:

           

        Просто и элегантно. И без долгих и утомительных вычислений. 🙂

        Именно поэтому операцию освобождения от иррациональности в дробях надо уметь делать. В подобных навороченных примерах только она и спасает, да.) Разумеется, внимательность никто не отменял. Бывают задания, где просят избавиться от иррациональности в числителе. Эти задания ничем от рассмотренных не отличаются, только от корней очищается числитель.)

Более сложные примеры

        Осталось рассмотреть некоторые специальные приёмы в работе с корнями и потренироваться распутывать не самые простые примеры. И тогда полученной информации уже будет достаточно для решения заданий с корнями любого уровня сложности. Итак — вперёд.) Для начала разберёмся, что делать со вложенными корнями, когда формула корня из корня не работает. Например, вот такой примерчик.

        Вычислить: 

        Корень под корнем… К тому же под корнями сумма или разность. Стало быть, формула корня из корня (с перемножением показателей) здесь не действует. Значит, надо что-то делать с подкоренными выражениями: у нас просто нету других вариантов. В таких примерах чаще всего под большим корнем зашифрован полный квадрат какой-нибудь суммы. Или разности. А корень из квадрата уже отлично извлекается! И теперь наша задача — его расшифровать.) Такая расшифровка красиво делается через систему уравнений. Сейчас всё сами увидите.)

        Итак, под первым корнем у нас вот такое выражение:

        А вдруг, не угадали? Проверим! Возводим в квадрат по формуле квадрата суммы:

        Всё верно.) Но… Откуда я взял это выражение ? С неба?

        Нет.) Мы его чуть ниже получим честно. Просто по данному выражению я показываю, как именно составители заданий шифруют такие квадраты. 🙂 Что такое 54? Это сумма квадратов первого и второго чисел. Причём, обратите внимание, уже без корней! А корень остаётся в удвоенном произведении, которое в нашем случае равно . Поэтому распутывание подобных примеров начинается с поиска удвоенного произведения. Если распутывать обычным подбором. И, кстати, о знаках. Тут всё просто. Если перед удвоенным плюс, то квадрат суммы. Если минус, то разности.) У нас плюс — значит, квадрат суммы.) А теперь — обещанный аналитический способ расшифровки. Через систему.)

        Итак, у нас под корнем явно тусуется выражение (a+b)2, и наша задача — найти a и b. В нашем случае сумма квадратов даёт 54. Вот и пишем:

        Теперь удвоенное произведение. Оно у нас . Так и записываем:

        Получили вот такую системку:

        Решаем обычным методом подстановки. Выражаем из второго уравнения, например,  и подставляем в первое:

        Решим первое уравнение:

           

           

       Получили биквадратное уравнение относительно a. Считаем дискриминант: 

       Значит,

       Получили аж четыре возможных значения a. Не пугаемся. Сейчас мы всё лишнее отсеем.) Если мы сейчас для каждого из четырёх найденных значений  посчитаем соответствующие значения , то получим четыре решения нашей системы. Вот они:

        И тут вопрос — а какое из решений нам подходит? Давайте подумаем. Отрицательные решения можно сразу отбросить: при возведении в квадрат минусы «сгорят», и всё подкоренное выражение в целом не изменится.) Остаются первые два варианта. Выбрать их можно совершенно произвольно: от перестановки слагаемых сумма всё равно не меняется.) Пусть, например, , а .

        Итого получили под корнем квадрат вот такой суммы:

        Всё чётко.)

        Я не зря так детально описываю ход решения. Чтобы было понятно, как происходит расшифровка.) Но есть одна проблемка. Аналитический способ расшифровки хоть и надёжный, но весьма длинный и громоздкий: приходится решать биквадратное уравнение, получать четыре решения системы и потом ещё думать, какие из них выбрать… Хлопотно? Согласен, хлопотно. Этот способ безотказно работает в большинстве подобных примеров. Однако очень часто можно здорово сократить себе работу и найти оба числа творчески. Подбором.) Да-да! Сейчас, на примере второго слагаемого (второго корня), я покажу более лёгкий и быстрый способ выделения полного квадрата под корнем.

        Итак, теперь у нас вот такой корень: .

        Размышляем так: «Под корнем — скорее всего, зашифрованный полный квадрат. Раз перед удвоенным минус — значит, квадрат разности. Сумма квадратов первого и второго чисел даёт нам число 54. Но какие это квадраты? 1 и 53? 49 и 5? Слишком много вариантов… Нет, лучше начать распутывать с удвоенного произведения. Наши  можно расписать как  . Раз произведение удвоенное, то двойку сразу отметаем. Тогда кандидатами на роль a и b остаются 7 и . А вдруг, это 14 и /2? Не исключено. Но начинаем-то всегда с простого!» Итак, пусть , а . Проверим их на сумму квадратов:

        Получилось! Значит, наше подкоренное выражение — это на самом деле квадрат разности:

 

        Вот такой вот способ-лайт, чтобы не связываться с системой. Не всегда работает, но во многих таких примерах его вполне достаточно. Итак, под корнями — полные квадраты. Осталось только правильно извлечь корни, да досчитать пример:

        А теперь разберём ещё более нестандартное задание на корни.)

        Докажите, что число A – целое, если .

        Впрямую ничего не извлекается, корни вложенные, да ещё и разных степеней… Кошмар! Однако, задание имеет смысл.) Стало быть, ключ к его решению имеется.) А ключ здесь такой. Рассмотрим наше равенство

как уравнение относительно A. Да-да! Хорошо бы избавиться от корней. Корни у нас кубические, поэтому возведём-ка обе части равенства в куб. По формуле куба суммы:

       Кубы и корни кубические друг друга компенсируют, а под каждым большим корнем забираем одну скобку у квадрата и сворачиваем произведение разности и суммы в разность квадратов:

        Отдельно сосчитаем разность квадратов под корнями:

        Отлично! Значит, всё наше равенство ещё сильнее упростится:

   

        А теперь делаем финт ушами — заменяем сумму корней в скобках на A (согласно условию примера!).

        Получаем кубическое уравнение  или .

        Здесь как раз тот случай, когда один из корней легко угадывается — это . Значит, наш многочлен  можно разложить как

        Как разложить? Либо по схеме Горнера, либо делением «уголком» на скобку (A-4), либо даже группировкой (если представить -3A как -16A+13A). Объяснять подробно деление уголком или схему Горнера в теме про корни — уже совсем отклоняться от курса.) Кто в теме — и так поймёт.

        А теперь легко заметить, что квадратный трёхчлен во вторых скобках имеет отрицательный дискриминант, а значит, наше уравнение имеет единственный действительный корень . И поэтому наша страшная сумма корней  в действительности равна просто 4. То есть, явно целому числу. Что и требовалось доказать.)

        А теперь — поупрощаем некоторые дробные выражения с корнями. От простого — к сложному. Здесь всё точно так же, как и с многочленами. Только в применении к корням.) Я же говорил, что действия с корнями ничем не отличаются от таковых с буквами. И к корням с таким же успехом применима вся алгебра седьмого класса — формулы сокращённого умножения, разложение на множители, приведение подобных и т.п.

        Например, такое задание.

        Сократить дробь:

        

        Пример явно намекает на применение формулы разности квадратов:

        Спрашивается, а где же здесь квадраты? Сплошные корни… Сейчас покажу. 🙂

        Берём числитель нашей дробушки: .

        Что такое ? По свойству корня из степени, мы можем вынести квадрат наружу. Вот так:

        Хорошо, а из  как квадрат сделать? Не вопрос! По пятому свойству, домножаем на двойку показатели корня и подкоренного выражения:

        По такой технологии, между прочим, можно совершенно любой корень превратить в совершенно любую степень. Какую хотим. 🙂  Как, например,  представить в виде 4-й степени? Нет проблем: 

        Хотим из степеней корни делаем, хотим — наоборот, степени из корней. Что хотим, то и творим. Математика, однако! 🙂

        Итак, весь наш числитель можно представить как разность квадратов:

        А дальше никаких проблем — раскладываем числитель на множители и сокращаем:

        Следующий пример.

        Упростить:

        

        Действуем аналогично. Раскладываем на множители и сокращаем. 🙂 В числителе применяем группировку. Например, вот такую:

        А в знаменателе просто выносим общий множитель :

        Подставляем всё в нашу дробь и сокращаем:

        Как видим, разложение на множители очень популярно в теме с корнями. Очень! И особенно — формула разности квадратов. Именно поэтому формулы сокращённого умножения так важно знать и уметь применять. 🙂

        Ну и на десерт распутаем что-нибудь навороченное. )

        Упростить:

        

        Чтобы не запутаться и не наляпать ошибок, будем действовать по порядку. При взгляде на любой пример всегда задаём сами себе вопрос: «Что в примере мне больше всего не нравится?» В данном примере большинство скажет: «Числитель первой дроби!» Верно! Вот и упростим его отдельно: остальная часть примера от этого никак не пострадает.) Итак,

        Вместо знака деления удобно использовать черту дроби. Вот так:

        Сначала упростим дробь. Как? Попробуем сократить.) Для этого, ясное дело, надо разложить на множители числитель и знаменатель, да… Берём отдельно числитель . Можно его разложить на множители? Можно! Для этого из a надо сделать корень. Вот так:

        Если теперь подставить вместо a выражение , то всплывёт общий множитель. 🙂

        Со знаменателем полная аналогия:

        Таким образом,

        Теперь от упрощённой дроби отнимаем единичку. Как? Делаем из единички дробь и — вперёд!

        Следующим пунктом идёт деление полученной дроби на выражение . Это означает, что оно пойдёт у нас в знаменатель:

        Уфф… Дальше… Отнимаем от полученного выражения дробь :

        И, наконец, последнее усилие. Возводим результат в куб:

        Ну как, всё понятно? Тогда — вперёд, набиваем руку и делаем примеры!

        Вычислить:

        Вынести множители за знак корня: ,  , где .

        Внести множители под знак корня: ,  .

        Освободиться от иррациональности в знаменателе дробей:

        , .

        Вычислить: 

        Доказать, что A – целое число, если .

        Упростить:

        Ответы (пока) давать не буду — иначе неинтересно. 🙂 До встречи и успехов!

Квадратный корень

Предварительные навыки

Основные сведения

Чтобы найти площадь квадрата, нужно длину его стороны возвести во вторую степень.

Найдём площадь квадрата, длина стороны которого 3 см

S = 32 = 9 см2

Теперь решим обратную задачу. А именно, зная площадь квадрата определим длину его стороны. Для этого воспользуемся таким инструментом как кóрень. Корень бывает квадратный, кубический, а также n-й степени.

Сейчас наш интерес вызывает квадратный корень. По другому его называют кóрнем второй степени.

Для нахождения длины стороны нашего квадрата, нужно найти число, вторая степень которого равна 9. Таковым является число 3. Это число и является кóрнем.

Введём для работы с корнями новые обозначения.

Символ кóрня выглядит как . Это по причине того, что слово корень в математике употребляется как радикал. А слово радикал происходит от латинского radix (что в переводе означает корень). Первая буква слова radix это r впоследствии преобразилась в символ корня .

Под корнем располагáют подкореннóе выражение. В нашем случае подкоренным выражением будет число 9 (площадь квадрата)

Нас интересовал квадратный корень (он же корень второй степени), поэтому слева над корнем указываем число 2. Это число называют показателем корня (или степенью корня)

Получили выражение, которое читается так: «квадратный корень из числа 9». С этого момента возникает новая задача по поиску самогó корня.

Если число 3 возвести во вторую степень, то получится число 9. Поэтому число 3 и будет ответом:

Значит квадрат площадью 9 см2 имеет сторону, длина которой 3 см. Приведённое действие называют извлечéнием квадрáтного кóрня.

Нетрудно догадаться, что квадратным корнем из числа 9 также является отрицательное число −3. При его возведении во вторую степень тоже получается число 9

Получается, что выражение  имеет два значения: 3 и −3. Но длина стороны квадрата не может быть отрицательным числом, поэтому для нашей задачи ответ будет только один, а именно 3.

Вообще, квадратный корень имеет два противоположных значения: положительное и отрицательное.

Например, извлечём квадратный корень из числа 4

Это выражение имеет два значения: 2 и −2, поскольку при возведении этих чисел во вторую степень, получится один и тот же результат 4

Поэтому ответ к выражению вида  записывают с плюсом и минусом. Плюс с минусом означает, что квадратный корень имеет два противоположных значения.

Запишем ответ к выражению  с плюсом и минусом:


Определения

Дадим определение квадратному корню.

Квадратным корнем из числа a называют такое число b, вторая степень которого равна a.

То есть число b должно быть таким, чтобы выполнялось равенство ba. Число b (оно же корень) обозначается через радикал  так, что . На практике левая и правая часть поменяны местами и мы видим привычное выражение 

Например, квадратным корнем из числá 16 есть число 4, поскольку число 4 во второй степени равно 16

42 = 16

Корень 4 можно обозначить через радикал  так, что .

Также квадратным корнем из числá 16 есть число −4, поскольку число −4 во второй степени равно 16

(−4)2 = 16

Если при решении задачи интересует только положительное значение, то корень называют не просто квадратным, а арифметическим квадратным.

Арифметический квадратный корень из числá a — это неотрицательное число b (b ≥ 0), при котором выполняется равенство ba.

В нашем примере квадратными корнями из числá 16 являются корни 4 и −4, но арифметическим из них является только корень 4.

В разговорном языке можно использовать сокращение. К примеру, выражение  полностью читается так: «квадратный корень из числá шестнадцать», а в сокращённом варианте можно прочитать так: «корень из шестнадцати».

Не следует путать понятия корень и квадрат. Квадрат это число, которое получилось в результате возведения какого-нибудь числá во вторую степень. Например, числа 25, 36, 49 являются квадратами, потому что они получились в результате возведения во вторую степень чисел 5, 6 и 7 соответственно.

Корнями же являются числа 5, 6 и 7. Они являются теми числами, которые во второй степени равны 25, 36 и 49 соответственно.

Чаще всего в квадратных корнях показатель кóрня вообще не указывается. Так, вместо записи можно использовать запись. Если в учебнике по математике встретится корень без показателя, то нужно понимать, что это квадратный корень.

Квадратный корень из единицы равен единице. То есть справедливо следующее равенство:

Это по причине того, что единица во второй степени равна единице:

12 = 1

и квадрат, состоящий из одной квадратной единицы, имеет сторону, равную единице:

Квадратный корень из нуля равен нулю. То есть справедливо равенство , поскольку 0= 0.

Выражение вида  смысла не имеет. Например, не имеет смысла выражение , поскольку вторая степень любого числа есть число положительное. Невозможно найти число, вторая степень которого будет равна −4.

Если выражение вида  возвести во вторую степень, то есть если записать , то это выражение будет равно подкореннóму выражению a

Например, выражение  равно 4

Это потому что выражение  равно значению 2. Но это значение сразу возвóдится во вторую степень и получается результат 4.

Еще примеры:

Корень из квадрата числá равен модулю этого числá:

Например, корень из числá 5, возведённого во вторую степень, равен модулю числá 5

Если во вторую степень возвóдится отрицательное число, ответ опять же будет положительным. Например, корень из числá −5, возведённого во вторую степень, равен модулю числá −5. А модуль числа −5 равен 5

Действительно, если не пользуясь правилом , вычислять выражение  обычным методом — сначала возвести число −5 во вторую степень, затем извлечь полученный результат, то полýчим ответ 5

Не следует путать правило  с правилом . Правило  верно при любом a, тогда как правило  верно в том случае, если выражение  имеет смысл.

В некоторых учебниках знак корня может выглядеть без верхней линии. Выглядит это так:

Примеры: √4, √9, √16.

Мéньшему числу соответствует мéньший корень, а бóльшему числу соответствует бóльший корень.

Например, рассмотрим числа 49 и 64. Число 49 меньше, чем число 64.

49 < 64

Если извлечь квадратные корни из этих чисел, то числу 49 будет соответствовать меньший корень, а числу 64 — бóльший. Действительно, √49 = 7, а √64 = 8,

√49 < √64

Отсюда:

7 < 8


Примеры извлечения квадратных корней

Рассмотрим несколько простых примеров на извлечение квадратных корней.

Пример 1. Извлечь квадратный корень √36

Данный квадратный корень равен числу, квадрат которого равен 36. Таковым является число 6, поскольку 6= 36

√36 = 6


Пример 2. Извлечь квадратный корень √49

Данный квадратный корень равен числу, квадрат которого равен 49. Таковым является число 7, поскольку 7= 49

√49 = 7

В таких простых примерах достаточно знать таблицу умножения. Так, мы помним, что число 49 входит в таблицу умножения на семь. То есть:

7 × 7 = 49

Но 7 × 7 это 72

7= 49

Отсюда, √49 = 7.


Пример 3. Извлечь квадратный корень √100

Данный квадратный корень равен числу, квадрат которого равен 100. Таковым является число 10, поскольку 102 = 100

√100 = 10

Число 100 это последнее число, корень которого можно извлечь с помощью таблицы умножения. Для чисел, бóльших 100, квадратные корни можно находить с помощью таблицы квадратов.


Пример 3. Извлечь квадратный корень √256

Данный квадратный корень равен числу, квадрат которого равен 256. Чтобы найти это число, воспользуемся таблицей квадратов.

Нахóдим в таблице квадратов число 256 и двигаясь от него влево и вверх определяем цифры, которые образуют число, квадрат которого равен 256.

Видим, что это число 16. Значит √256 = 16.


Пример 4. Найти значение выражения 2√16

В данном примере число 2 умножается на выражение с корнем. Сначала вычислим корень √16, затем перемнóжим его с числом 2


Пример 7. Решить уравнение 

В данном примере нужно найти значение переменной x, при котором левая часть будет равна 4.

Значение переменной x равно 16, поскольку . Значит корень уравнения равен 16.

Примечание. Не следует путать корень уравнения и квадратный корень. Корень уравнения это значение переменной, при котором уравнение обращается в верное числовое равенство. А квадратный корень это число, вторая степень которого равна выражению, находящемуся под радикалом .

Подобные примеры решают, пользуясь определением квадратного корня. Давайте и мы поступим так же.

Из определения мы знаем, что квадратный корень  равен числу b, при котором выполняется равенство ba.

Применим равенство ba к нашему примеру . Роль переменной b у нас играет число 4, а роль переменной a — выражение, находящееся под корнем , а именно переменная x

В выражении 4x вычислим левую часть, полýчим 16 = x. Поменяем левую и правую часть местами, полýчим = 16. В результате приходим к тому, что нашлось значение переменной x.


Пример 8. Решить уравнение 

Перенесем −8 в правую часть, изменив знак:

Возведем правую часть во вторую степень и приравняем её к переменной x

Вычислим правую часть, полýчим 64 = x. Поменяем левую и правую часть местами, полýчим = 64. Значит корень уравнения  равен 64


Пример 9. Решить уравнение 

Воспользуемся определением квадратного корня:

Роль переменной b играет число 7, а роль переменной a — подкореннóе выражение 3 + 5x. Возведем число 7 во вторую степень и приравняем его к 3 + 5x

В выражении 72 = 3 + 5x вычислим левую часть полýчим 49 = 3 + 5x. Получилось обычное линейное уравнение. Решим его:

Корень уравнения  равен . Выполним проверку, подставив его в исходное уравнение:


Пример 10. Найти значение выражения 

В этом выражении число 2 умножается на квадратный корень из числа 49.

Сначала нужно извлечь квадратный корень и перемножить его с числом 2


Приближённое значение квадратного корня

Не каждый квадратный корень можно извлечь. Извлечь квадратный корень можно только в том случае, если удаётся найти число, вторая степень которого равна подкореннóму выражению.

Например, извлечь квадратный корень  можно, потому что удаётся найти число, вторая степень которого равна подкореннóму выражению. Таковым является число 8, поскольку 8= 64. То есть

А извлечь квадратный корень  нельзя, потому что невозможно найти число, вторая степень которого равна 3. В таком случае говорят, что квадратный корень из числа 3 не извлекается.

Зато можно извлечь квадратный корень из числа 3 приближённо. Извлечь квадратный корень приближённо означает найти значение, которое при возведении во вторую степень будет максимально близко к подкореннóму выражению.

Приближённое значение ищут с определенной точностью: с точностью до целых, с точностью до десятых, с точностью до сотых и так далее.

Найдём значение корня  приближённо с точностью до десятых. Словосочетание «с точностью до десятых» говорит о том, что приближённое значение корня  будет представлять собой десятичную дробь, у которой после запятой одна цифра.

Для начала найдём ближайшее меньшее число, корень которого можно извлечь. Таковым является число 1. Корень из этого числа равен самому этому числу:

√1 = 1

Аналогично находим ближайшее бóльшее число, корень которого можно извлечь. Таковым является число 4. Корень из этого числа равен 2

√4 = 2

√1 меньше, чем √4

√1 < √4

А √3 больше, чем √1 но меньше, чем √4. Запишем это в виде двойного неравенства:

√1 < √3 < √4

Точные значения корней √1 и √4 известны. Это числа 1 и 2

1 < √3 < 2

Тогда очевидно, что значение корня √3 будет представлять собой десятичную дробь, потому что между числами 1 и 2 нет целых чисел.

Для нахождения приближённого значения квадратного корня √3 будем проверять десятичные дроби, располагающиеся в интервале от 1 до 2, возводя их в квадрат. Делать это будем до тех пор пока не полýчим значение, максимально близкое к 3. Проверим к примеру дробь 1,1

1,12 = 1,21

Получился результат 1,21, который не очень близок к подкореннóму выражению 3. Значит 1,1 не годится в качестве приближённого значения квадратного корня √3, потому что оно малó.

Проверим тогда дробь 1,8

1,82 = 3,24

Получился результат 3,24, который близок к подкореннóму выражению, но превосходит его на 0,24. Значит 1,8 не годится в качестве приближенного значения корня √3, потому что оно великó.

Проверим тогда дробь 1,7

1,72 = 2,89

Получился результат 2,89, который уже близок к подкореннóму выражению. Значит 1,7 и будет приближённым значением квадратного корня √3. Напомним, что знак приближенного значения выглядит как ≈

√3 ≈ 1,7

Значение 1,6 проверять не нужно, потому что в результате получится число 2,56, которое дальше от трёх, чем значение 2,89. А значение 1,8, как было показано ранее, является уже большим.

В данном случае мы нашли приближенное значение корня √3 с точностью до десятых. Значение можно получить ещё более точно. Для этого его следует находить с точностью до сотых.

Чтобы найти значение с точностью до сотых проверим десятичные дроби в интервале от 1,7 до 1,8

1,7 < √3 < 1,8

Проверим дробь 1,74

1,742 = 3,0276

Получился результат 3,0276, который близок к подкореннóму выражению, но превосходит его на 0,0276. Значит значение 1,74 великó для корня √3.

Проверим тогда дробь 1,73

1,732 = 2,9929

Получился результат 2,9929, который близок к подкореннóму выражению √3. Значит 1,73 будет приближённым значением квадратного корня √3 с точностью до сотых.

Процесс нахождения приближённого значения квадратного корня продолжается бесконечно. Так, корень √3 можно находить с точностью до тысячных, десятитысячных и так далее:

√3 = 1,732 (вычислено с точностью до тысячных)

√3 = 1,7320 (вычислено с точностью до десятитысячных)

√3 = 1,73205 (вычислено с точностью до ста тысячных).

Ещё квадратный корень можно извлечь с точностью до целых. Приближённое значение квадратного корня √3 с точностью до целых равно единице:

√3 ≈ 1

Значение 2 будет слишком большим, поскольку при возведении этого числа во вторую степень получается число 4, которое больше подкоренного выражения. Нас же интересуют значения, которые при возведении во вторую степень равны подкореннóму выражению или максимально близки к нему, но не превосходят его.

В зависимости от решаемой задачи допускается находить значение, вторая степень которого больше подкоренного выражения. Это значение называют приближённым значением квадратного корня с избытком. Поговорим об этом подробнее.


Приближенное значение квадратного корня с недостатком или избытком

Иногда можно встретить задание, в котором требуется найти приближённое значение корня с недостатком или избытком.

В предыдущей теме мы нашли приближённое значение корня √3 с точностью до десятых с недостатком. Недостаток понимается в том смысле, что до значения 3 нам недоставало ещё некоторых частей. Так, найдя приближённое значение √3 с точностью до десятых, мы получили 1,7. Это значение является значением с недостатком, поскольку при возведении этого числа во вторую степень полýчим результат 2,89. Этому результату недостаёт ещё 0,11 чтобы получить число 3. То есть, 2,89 + 0,11 = 3.

С избытком же называют приближённые значения, которые при возведении во вторую степень дают результат, который превосходит подкореннóе выражение. Так, вычисляя корень √3 приближённо, мы проверили значение 1,8. Это значение является приближённым значением корня √3 с точностью до десятых с избытком, поскольку при возведении 1,8 во вторую степень, получаем число 3,24. Этот результат превосходит подкореннóе выражение на 0,24. То есть 3,24 − 3 = 0,24.

Приближённое значение квадратного корня √3 с точностью до целых тоже был найден с недостатком:

√3 ≈ 1

Это потому что при возведении единицы в квадрат получаем единицу. То есть до числа 3 недостаёт ещё 2.

Приближённое значение квадратного корня √3 с точностью до целых можно найти и с избытком. Тогда этот корень приближённо будет равен 2

√3 ≈ 2

Это потому что при возведении числа 2 в квадрат получаем 4. Число 4 превосходит подкореннóе выражение 3 на единицу. Извлекая приближённо квадратный корень с избытком желательно уточнять, что корень извлечен именно с избытком:

√3 ≈ 2 (с избытком)

Потому что приближённое значение чаще всего ищется с недостатком, чем с избытком.

Дополнительно следует упомянуть, что в некоторых учебниках словосочетания «с точностью до целых», «с точностью до десятых», с «точностью до сотых», заменяют на словосочетания «с точностью до 1», «с точностью до 0,1», «с точностью до 0,01» соответственно.

Так, если в задании сказано извлечь квадратный корень из числа 5 с точностью до 0,01, то это значит что корень следует извлекать приближённо с точностью до сотых:

√5 ≈ 2,23


Пример 2. Извлечь квадратный корень из числа 51 с точностью до 1

√51 ≈ 7


Пример 3. Извлечь квадратный корень из числа 51 с точностью до 0,1

√51 ≈ 7,1

Пример 4. Извлечь квадратный корень из числа 51 с точностью до 0,01

√51 ≈ 7,14


Границы, в пределах которых располагаются корни

Если исходное число принадлежит промежутку [1; 100], то квадратный корень из этого исходного числа будет принадлежать промежутку [1; 10].

Например, пусть исходным числом будет 64. Данное число принадлежит промежутку [1; 100]. Сразу делаем вывод, что квадратный корень из числа 64 будет принадлежать промежутку [1; 10]. Теперь вспоминаем таблицу умножения. Какое перемножение двух одинаковых сомножителей даёт в результате 64? Ясно, что перемножение 8 × 8, а это есть 8= 64. Значит квадратный корень из числа 64 есть 8


Пример 2. Извлечь квадратный корень из числа 49

Число 49 принадлежит промежутку [1; 100]. Значит квадратный корень будет принадлежать промежутку [1; 10]. Этим корнем будет число 7, поскольку 7= 49

√49 = 7


Пример 2. Извлечь квадратный корень из числа 1

Число 1 принадлежит промежутку [1; 100]. Значит квадратный корень будет принадлежать промежутку [1; 10]. Этим корнем будет число 1, поскольку 1= 1

√1 = 1


Пример 3. Извлечь квадратный корень из числа 100

Число 100 принадлежит промежутку [1; 100]. Значит квадратный корень будет принадлежать промежутку [1; 10]. Этим корнем будет число 10, поскольку 10= 100

√100 = 10

Понятно, что промежуток [1; 100] содержит ещё и числа, квадратные корни из которых не извлекаются. Для таких чисел корень нужно извлекать приближённо. Тем не менее, приближённый корень тоже будет располагаться в пределах промежутка [1; 10].

Например, извлечём квадратный корень из числа 37. Нет целого числа, вторая степень которого была бы равна 37. Поэтому извлекать квадратный корень следует приближённо. Извлечём его к примеру с точностью до сотых:

√37 ≈ 6,08

Для облегчения можно находить ближайшее меньшее число, корень из которого извлекается. Таковым в данном примере было число 36. Квадратный корень из него равен 6. И далее отталкиваясь от числа 6, можно находить приближённое значение корня √37, проверяя различные десятичные дроби, целая часть которых равна 6.

Квадраты чисел от 1 до 10 обязательно нужно знать наизусть. Ниже представлены эти квадраты:

12 = 1
22 = 4
32 = 9
42 = 16
52 = 25
62 = 36
72 = 49
82 = 64
92 = 81
102 = 100

И обратно, следует знать значения квадратных корней этих квадратов:

Если к любому числу от 1 до 10 в конце дописать ноль (или несколько нулей), и затем возвести это число во вторую степень, то в полученном числе будет в два раза больше нулей.

Например, 6= 36. Допишем к числу 6 один ноль, полýчим 60. Возведём число 60 во вторую степень, полýчим 3600

60= 3600

А если к числу 6 дописать два нуля, и возвести это число во вторую степень, то полýчим число, в котором четыре нуля. То есть в два раза больше нулей:

6002 = 360000

Тогда можно сделать следующий вывод:

Если исходное число содержит знакомый нам квадрат и чётное количество нулей, то можно извлечь квадратный корень из этого числа. Для этого следует извлечь корень из знакомого нам квадрата и затем записать половину количества нулей из исходного числа.

Например, извлечём квадратный корень из числа 900. Видим, что в данном числе есть знакомый нам квадрат 9. Извлекаем из него корень, получаем 3

Теперь из исходного числа записываем половину от количества нулей. В исходном числе 900 содержится два нуля. Половина этого количества нулей есть один ноль. Записываем его в ответе после цифры 3


Пример 2. Извлечём квадратный корень из числа 90000

Здесь опять же имеется знакомый нам квадрат 9 и чётное количество нулей. Извлекаем корень из числа 9 и записываем половину от количества нулей. В исходном числе содержится четыре нуля. Половиной же этого количества нулей будет два нуля:


Пример 3. Извлечем квадратный корень из числа 36000000

Здесь имеется знакомый нам квадрат 36 и чётное количество нулей. Извлекаем корень из числа 36 и записываем половину от количества нулей. В исходном числе шесть нулей. Половиной же будет три нуля:


Пример 4. Извлечем квадратный корень из числа 2500

Здесь имеется знакомый нам квадрат 25 и чётное количество нулей. Извлекаем корень из числа 25 и записываем половину от количества нулей. В исходном числе два нуля. Половиной же будет один ноль:


Если подкореннóе число увеличить (или уменьшить) в 100, 10000 то корень увеличится (или уменьшится) в 10, 100 раз соответственно.

Например, . Если увеличим подкореннóе число в 100 раз, то квадратный корень увеличится в 10 раз:

И наоборот, если в равенстве  уменьшим подкореннóе число в 100 раз, то квадратный корень уменьшится в 10 раз:

Пример 2. Увеличим в равенстве  подкореннóе число в 10000, тогда квадратный корень 70 увеличиться в 100 раз

Пример 3. Уменьшим в равенстве  подкореннóе число в 100 раз, тогда квадратный корень 70 уменьшится в 10 раз

Эта закономерность позволяет извлечь квадратный корень из десятичной дроби, если в данной дроби после запятой содéржатся две цифры, и эти две цифры образуют знакомый нам квадрат. В таких случаях данную десятичную дробь следует умножить на 100. Затем извлечь квадратный корень из получившегося числа и уменьшить подкореннóе число в сто раз.

Например, извлечём квадратный корень из числа 0,25. В данной десятичной дроби после запятой содержатся две цифры и эти две цифры образуют знакомый нам квадрат 25.

Умнóжим десятичную дробь 0,25 на 100, полýчим 25. А из числа 25 квадратный корень извлекается легко:

Но нам изначально нужно было извлечь корень из 0,25, а не из 25. Чтобы исправить ситуацию, вернём нашу десятичную дробь. Если в равенстве  подкореннóе число уменьшить в 100 раз, то полýчим под корнем 0,25 и соответственно ответ уменьшится в 10 раз:

Обычно в таких случаях достаточно уметь передвигáть запятую. Потому что сдвинуть в числе запятую вправо на две цифры это всё равно что умножить это число на 100.

В предыдущем примере в подкоренном числе 0,25 можно было сдвинуть запятую вправо на две цифры, а в полученном ответе сдвинуть её влево на одну цифру.

Например, извлечем корень из числа 0,81. Мысленно передвинем запятую вправо на две цифры, полýчим 81. Теперь извлечём квадратный корень из числа 81, полýчим ответ 9. В ответе 9 передвинем запятую влево на одну цифру, полýчим 0,9. Значит, .

Это правило работает и в ситуации, когда после запятой содержатся четыре цифры и эти цифры образуют знакомый нам квадрат.

Например, десятичная дробь 0,1225 содержит после запятой четыре цифры. Эти четыре цифры образуют число 1225, квадратный корень из которого равен 35.

Тогда можно извлечь квадратный корень и из 0,1225. Умнóжим данную десятичную дробь на 10000, полýчим 1225. Из числа 1225 квадратный корень можно извлечь с помощью таблицы квадратов:

Но нам изначально нужно было извлечь корень из 0,1225, а не из 1225. Чтобы исправить ситуацию, в равенстве  подкореннóе число уменьшим в 10000 раз. В результате под корнем образуется десятичная дробь 0,1225, а правая часть уменьшится в 100 раз

Эта же закономерность будет работать и при извлечении корней из дробей вида 12,25. Если цифры из которых состоит десятичная дробь образуют знакомый нам квадрат, при этом после запятой содержится чётное количество цифр, то можно извлечь корень из этой десятичной дроби.

Умнóжим десятичную дробь 12,25 на 100, полýчим 1225. Извлечём корень из числа 1225

Теперь в равенстве уменьшим подкореннóе число в 100 раз. В результате под корнем образуется число 12,25, и соответственно ответ уменьшится в 10 раз


Если исходное число принадлежит промежутку [100; 10000], то квадратный корень из этого исходного числа будет принадлежать промежутку [10; 100].

В этом случае применяется таблица квадратов:

Например, пусть исходным числом будет 576. Данное число принадлежит промежутку [100; 10000]. Сразу делаем вывод, что квадратный корень из числа 576 будет принадлежать промежутку [10; 100]. Теперь открываем таблицу квадратов и смотрим какое число во второй степени равно 576

Видим, что это число 24. Значит .


Пример 2. Извлечь квадратный корень из числа 432.

Число 432 принадлежит промежутку [100; 10000]. Значит квадратный корень следует искать в промежутке [10; 100]. Открываем таблицу квадратов и смотрим какое число во второй степени равно 432. Обнаруживаем, что число 432 в таблице квадратов отсутствует. В этом случае квадратный корень следует искать приближённо.

Извлечем квадратный корень из числа 432 с точностью до десятых.

В таблице квадратов ближайшее меньшее число к 432 это число 400. Квадратный корень из него равен 20. Отталкиваясь от числа 20, будем проверять различные десятичные дроби, целая часть которых равна 20.

Проверим, например, число 20,8. Для этого возведём его в квадрат:

20,82 = 432,64

Получилось число 432,64 которое превосходит исходное число 432 на 0,64. Видим, что значение 20,8 великó для корня √432. Проверим тогда значение 20,7

20,7= 428,49

Значение 20,7 годится в качестве корня, поскольку в результате возведения этого числа в квадрат получается число 428,49, которое меньше исходного числа 432, но близко к нему. Значит √432 ≈ 20,7.

Необязательно запоминать промежутки чтобы узнать в каких границах располагается корень. Можно воспользоваться методом нахождения ближайших квадратов с чётным количеством нулей на конце.

Например, извлечём корень из числа 4225. Нам известен ближайший меньший квадрат 3600, и ближайший больший квадрат 4900

3600 < 4225 < 4900

Извлечём квадратные корни из чисел 3600 и 4900. Это числа 60 и 70 соответственно:

Тогда можно понять, что квадратный корень из числа 4225 располагается между числами 60 и 70. Можно даже найти его методом подбора. Корни 60 и 70 исключаем сразу, поскольку это корни чисел 3600 и 4900. Затем можно проверить, например, корень 64. Возведём его в квадрат (или умнóжим данное число само на себя)

Корень 64 не годится. Проверим корень 65

Получается 4225. Значит 65 является корнем числа 4225


Тождественные преобразования с квадратными корнями

Над квадратными корнями можно выполнять различные тождественные преобразования, тем самым облегчая их вычисление. Рассмотрим некоторые из этих преобразований.

Квадратный корень из произведения

Квадратный корень из произведения это выражение вида , где a и b некоторые числа.

Например, выражение  является квадратным корнем из произведения чисел 4 и 9.

Чтобы извлечь такой квадратный корень, нужно по отдельности извлечь квадратные корни из множителей 4 и 9, представив выражение  в виде произведения корней . Вычислив по отдельности эти корни полýчим произведение 2 × 3, которое равно 6

Конечно, можно не прибегать к таким манипуляциям, а вычислить сначала подкореннóе выражение 4 × 9, которое равно 36. Затем извлечь квадратный корень из числа 36

Но при извлечении квадратных корней из больших чисел это правило может оказаться весьма полезным.

Допустим, потребовалось извлечь квадратный корень из числа 144. Этот корень легко определяется с помощью таблицы квадратов — он равен 12

Но предстáвим, что таблицы квадратов под рукой не оказалось. В этом случае число 144 можно разложить на простые множители. Затем из этих простых множителей составить числа, квадратные корни из которых извлекаются.

Итак, разлóжим число 144 на простые множители:

Получили следующее разложение:

В разложéнии содержатся четыре двойки и две тройки. При этом все числа, входящие в разложение, перемнóжены. Это позволяет предстáвить произведения одинаковых сомножителей в виде степени с показателем 2.

Тогда четыре двойки можно заменить на запись 2× 22, а две тройки заменить на 32

В результате будем иметь следующее разложение:

Теперь можно извлекáть квадратный корень из разложения числа 144

Применим правило извлечения квадратного корня из произведения:

Ранее было сказано, что если подкореннóе выражение возведенó во вторую степень, то такой квадратный корень равен модулю из подкореннóго выражения.

Тогда получится произведение 2 × 2 × 3, которое равно 12

Простые множители представляют в виде степени для удобства и короткой записи. Допускается также записывать их под кóрнем как есть, чтобы впоследствии перемнóжив их, получить новые сомножители.

Так, разложив число 144 на простые множители, мы получили разложение 2 × 2 × 2 × 2 × 3 × 3. Это разложение можно записать под кóрнем как есть:

затем перемнóжить некоторые сомножители так, чтобы получились числа, квадратные корни из которых извлекаются. В данном случае можно дважды перемнóжить две двойки и один раз перемнóжить две тройки:

Затем применить правило извлечения квадратного корня из произведения и получить окончательный ответ:

С помощью правила извлечения квадратного корня из произведения можно извлекать корень и из других больших чисел. В том числе, из тех чисел, которых нет в таблице квадратов.

Например, извлечём квадратный корень из числа 13456. Этого числа нет в таблице квадратов, поэтому воспользуемся правилом извлечения квадратного корня из произведения, предварительно разложив число 13456 на простые множители.

Итак, разложим число 13456 на простые множители:

В разложении имеются четыре двойки и два числа 29. Двойки дважды предстáвим как 22. А два числа 29 предстáвим как 292. В результате полýчим следующее разложение числа 13456

Теперь будем извлекать квадратный корень из разложения числа 13456

Итак, если ≥ 0 и ≥ 0, то . То есть корень из произведения неотрицательных множителей равен произведению корней из этих множителей.

Докажем равенство . Для этого воспользуемся определением квадратного корня.

Согласно определению, квадратным корня из числа a есть число b, при котором выполняется равенство b= a.

В нашем случае нужно удостовериться, что правая часть равенства  при возведении во вторую степень даст в результате подкореннóе выражение левой части, то есть выражение ab.

Итак, выпишем правую часть равенства  и возведём ее во вторую степень:

Теперь воспользуемся правилом возведения в степень произведения. Согласно этому правилу, каждый множитель данного произведения нужно возвести в указанную степень:

Ранее было сказано, что если выражение вида  возвести во вторую степень, то получится подкореннóе выражение. Применим это правило. Тогда полýчим ab. А это есть подкореннóе выражение квадратного корня

Значит равенство  справедливо, поскольку при возведéнии правой части во вторую степень, получается подкореннóе выражение левой части.

Правило извлечения квадратного корня из произведения работает и в случае, если под кóрнем располагается более двух множителей. То есть справедливым будет следующее равенство:

, при ≥ 0 и ≥ 0, ≥ 0.


Пример 1. Найти значение квадратного корня 

Запишем корень в виде произведения корней, извлечём их, затем найдём значение полученного произведения:


Пример 2. Найти значение квадратного корня 

Предстáвим число 250 в виде произведения чисел 25 и 10. Делать это будем под знáком корня:

Теперь под кóрнем образовалось два одинаковых множителя 10 и 10. Перемнóжим их, полýчим 100

Далее применяем правило извлечения квадратного кóрня из произведения и получáем окончательный ответ:


Пример 3. Найти значение квадратного корня 

Воспользуемся правилом возведения степени в степень. Степень 114 предстáвим как (112)2.

Теперь воспользуемся правилом извлечения квадратного кóрня из квадрата числа:

В нашем случае квадратный корень из числа (112)2 будет равен 112. Говоря простым языком, внешний показатель степени 2 исчезнет, а внутренний останется:

Далее возводим число 11 во вторую степень и получаем окончательный ответ:

Этот пример также можно решить, воспользовавшись правилом извлечения квадратного корня из произведения. Для этого подкореннóе выражение 114 нужно записать в виде произведения 11× 112. Затем извлечь квадратный корень из этого произведения:


Пример 4. Найти значение квадратного корня

Перепишем степень 34 в виде (32)2, а степень 56 в виде (53)2

Далее используем правило извлечения квадратного кóрня из произведения:

Далее используем правило извлечения квадратного кóрня из квадрата числа:

Вычислим произведение получившихся степеней и полýчим окончательный ответ:


Сомножители, находящиеся под корнем, могут быть десятичными дробями. Например, извлечём квадратный корень из произведения

Запишем корень  в виде произведения корней, извлечём их, затем найдём значение полученного произведения:


Пример 6. Найти значение квадратного корня


Пример 7. Найти значение квадратного корня


Если первый сомножитель умножить на число n, а второй сомножитель разделить на это число n, то произведение не изменится.

Например, произведение 8 × 4 равно 32

8 × 4 = 32

Умнóжим сомножитель 8 скажем на число 2, а сомножитель 4 раздéлим на это же число 2. Тогда получится произведение 16 × 2, которое тоже равно 32.

(8 × 2) × (4 : 2) = 32

Это свойство полезно при решении некоторых задач на извлечение квадратных корней. Сомножители подкореннóго выражения можно умнóжить и разделить так, чтобы корни из них извлекались.

Например, извлечём квадратный корень из произведения . Если сразу воспользоваться правилом извлечения квадратного корня из произведения, то не полýчится извлечь корни √1,6 и √90, потому что они не извлекаются.

Проанализировав подкореннóе выражение 1,6 × 90, можно заметить, что если первый сомножитель 1,6 умножить на 10, а второй сомножитель 90 разделить на 10, то полýчится произведение 16 × 9. Из такого произведения квадратный корень можно извлечь, пользуясь правилом извлечения квадратного корня из произведения.

Запишем полное решение данного примера:

Процесс умножения и деления можно выполнять в уме. Также можно пропустить подробную запись извлечения квадратного корня из каждого сомножителя. Тогда решение станóвится короче:


Пример 9. Найти значение квадратного корня

Умнóжим первый сомножитель на 10, а второй раздéлим на 10. Тогда под кóрнем образуется произведение 36 × 0,04, квадратный корень из которого извлекается:


Если в равенстве поменять местами левую и правую часть, то полýчим равенство . Это преобразовáние позволяет упрощáть вычисление некоторых корней.

Например, узнáем чему равно значение выражения .

Квадратные корни из чисел 10 и 40 не извлекаются. Воспользуемся правилом , то есть заменим выражение из двух корней  на выражение с одним корнем, под которым будет произведение из чисел 10 и 40

Теперь найдём значение произведения, находящегося под корнем:

А квадратный корень из числа 400 извлекается. Он равен 20

Сомножители, располагáющиеся под корнем, можно расклáдывать на множители, группировáть, представлять в виде степени, а также перемножáть для получения новых сомножителей, корни из которых извлекаются.

Например, найдём значение выражения .

Воспользуемся правилом

Сомножитель 32 это 25. Предстáвим этот сомножитель как 2 × 24

Перемнóжим сомножители 2 и 2, полýчим 4. А сомножитель 24 предстáвим в виде степени с показателем 2

Теперь воспóльзуемся правилом и вычислим окончательный ответ:


Пример 12. Найти значение выражения

Воспользуемся правилом

Сомножитель 8 это 2 × 2 × 2, а сомножитель 98 это 2 × 7 × 7

Теперь под кóрнем имеются четыре двойки и две семёрки. Четыре двойки можно записать как 2× 22, а две семёрки как 72

Теперь воспользуемся правилом и вычислим окончательный ответ:


Квадратный корень из дроби

Квадратный корень вида равен дроби, в числителе которой квадратный корень из числа a, а в знаменателе — квадратный корень из числа b

Например, квадратный корень из дроби  равен дроби, в числителе которой квадратный корень из числа 4, а в знаменателе — квадратный корень из числа 9

Вычислим квадратные корни в числителе и знаменателе:

Значит, квадратный корень из дроби равен .

Докáжем, что равенство является верным.

Возведём правую часть во вторую степень. Если в результате полýчим дробь , то это будет означать, что равенство верно:


Пример 1. Извлечь квадратный корень 

Воспользуемся правилом извлечения квадратного корня из дроби:


Пример 2. Извлечь квадратный корень 

Переведём подкореннóе выражение в неправильную дробь, затем воспользуемся правилом извлечения квадратного корня из дроби:


Пример 3. Извлечь квадратный корень

Квадратным корнем из числа 0,09 является 0,3. Но можно извлечь этот корень, воспользовавшись правилом извлечения квадратного корня из дроби.

Предстáвим подкоренное выражение в виде обыкновенной дроби. 0,09 это девять сотых:

Теперь можно воспользоваться правилом извлечения квадратного корня из дроби:


Пример 4. Найти значение выражения 

Извлечём корни из 0,09 и 0,25, затем сложим полученные результаты:

Также можно воспользоваться правилом извлечения квадратного корня из дроби:

В данном примере первый способ оказался проще и удобнее.


Пример 5. Найти значение выражения 

Сначала вычислим квадратный корень, затем перемнóжим его с 10. Получившийся результат вычтем из 4


Пример 6. Найти значение выражения 

Сначала найдём значение квадратного корня . Он равен 0,6 поскольку 0,6= 0,36

Теперь вычислим получившееся выражение. Согласно порядку действий, сначала надо выполнить умножение, затем сложение:


Вынесение множителя из-под знака корня

В некоторых задачах может быть полезным вынесение множителя из-под знака корня.

Рассмотрим квадратный корень из произведения . Согласно правилу извлечения квадратного корня из произведения, нужно извлечь квадратный корень из каждого множителя данного произведения:

В нашем примере квадратный корень извлекается только из множителя 4. Его мы извлечём, а выражение  оставим без изменений:

Это и есть вынесение множителя из-под знака корня.

На практике подкореннóе выражение чаще всего требуется разложить на множители.


Пример 2. Вынести множитель из-под знака корня в выражении

Разлóжим подкореннóе выражение на множители 9 и 2. Тогда полýчим:

Теперь воспользуемся правило извлечения квадратного корня из произведения. Извлечь можно только корень из множителя 9. Множитель 2 остáвим под кóрнем:


Пример 3. Вынести множитель из-под знака корня в выражении

Разлóжим подкореннóе выражение на множители 121 и 3. Тогда полýчим:

Теперь воспользуемся правилом извлечения квадратного корня из произведения. Извлечь можно только корень из множителя 121. Выражение √3 остáвим под корнем:


Пример 4. Вынести множитель из-под знака корня в выражении

Воспользуемся правилом извлечения квадратного корня из произведения:

Квадратный корень извлекается только из числа 121. Извлечём его, а выражение √15 оставим без изменений:

Получается, что множитель 11 вынесен из-под знака корня. Вынесенный множитель принято записывать до выражения с корнем. Поменяем выражения √15 и 11 местами:


Пример 5. Вынести множитель из-под знака корня в выражении

Разлóжим подкореннóе выражение на множители 4 и 3

Воспользуемся правилом извлечения квадратного корня из произведения:

Извлечём корень из числа 4, а выражение √3 остáвим без изменений:


Пример 6. Упростить выражение 

Предстáвим второе слагаемое в виде . А третье слагаемое предстáвим в виде

Теперь в выражениях и вынесем множитель из-под знака корня:

Во втором слагаемом перемнóжим числа −4 и 4. Остальное перепишем без изменений:

Замечáем, что получившемся выражении квадратный корень √3 является общим множителем. Вынесем его за скобки:

Вычислим содержимое скобок, полýчим −1

Если множителем является −1, то записывают только минус. Единица опускается. Тогда полýчим окончательный ответ −√3


Внесение множителя под знак корня

Рассмотрим следующее выражение:

В этом выражении число 5 умнóжено на квадратный корень из числа 9. Найдём значение этого выражения.

Сначала извлечём квадратный корень, затем перемнóжим его с числом 5.

Квадратный корень из 9 равен 3. Перемнóжим его с числом 5. Тогда полýчим 15

Число 5 в данном случае было множителем. Внесём этот множитель под знак корня. Но сделать это нужно таким образом, чтобы в результате наших действий значение исходного выражения не изменилось. Проще говоря, после внесения множителя 5 под знак корня, получившееся выражение по-прежнему должно быть равно 15.

Значение выражения не изменится, если число 5 возвести во вторую степень и только тогда внести его под корень:

Итак, если данó выражение , и нужно внести множитель a под знак корня, то надо возвести во вторую степень множитель a и внести его под корень:

Пример 1. Внести множитель под знак корня в выражении

Возведём число 7 во вторую степень и внесём его под знак корня:


Пример 2. Внести множитель под знак корня в выражении 

Возведём число 10 во вторую степень и внесем его под знак корня:


Пример 3. Внести множитель под знак корня в выражении 

Вносить под знак корня можно только положительный множитель. Ранее было сказано, что выражение вида  не имеет смысла.

Однако, если перед знаком кóрня располагается отрицательный множитель, то минус можно оставить за знáком корня, а самó число внести под знак корня.

Пример 4. Внести множитель по знак корня в выражении 

В этом примере под знак корня внóсится только 3. Минус остаётся за знáком корня:


Пример 5. Выполнить возведéние в степень в следующем выражении:

Воспользуемся формулой квадрата суммы двух выражений:

(a + b)2 = a+ 2ab + b2

Роль переменной a в данном случае играет выражение √3, роль переменной b — выражение √2. Тогда полýчим:

Теперь необходимо упростить получившееся выражение.

Для выражений и  применим правило . Ранее мы говорили, что если выражение вида  возвести во вторую степень, то это выражение будет равно подкореннóму выражению a.

А в выражении для множителей и применим правило . То есть заменим произведение корней на один общий корень:

Приведём подобные слагаемые. В данном случае можно сложить слагаемые 3 и 2. А в слагаемом вычислить произведение, которое под кóрнем:


 

Задания для самостоятельного решения

Задание 1. Найдите значение квадратного корня:

Решение:

Задание 2. Найдите значение квадратного корня:

Решение:

Задание 3. Найдите значение квадратного корня:

Решение:

Задание 4. Найдите значение выражения:

Решение:

Задание 5. Найдите значение квадратного корня:

Решение:

Задание 6. Найдите значение квадратного корня:

Решение:

Задание 7. Найдите значение квадратного корня:

Решение:

Задание 8. Найдите значения следующих выражений:

Решение:

Задание 9. Извлеките квадратный корень из числа 4624

Решение:

Задание 10. Извлеките квадратный корень из числа 11025

Решение:

Задание 11. Найдите значение квадратного корня:

Решение:

Задание 12. Найдите значение квадратного корня:

Решение:

Задание 13. Найдите значение квадратного корня:

Решение:

Задание 14. Найдите значение квадратного корня:

Решение:

Задание 15. Найдите значение квадратного корня:

Решение:

Задание 16. Найдите значение выражения:

Решение:

Задание 17. Найдите значение выражения:

Решение:

Задание 18. Найдите значение выражения:

Решение:

Задание 19. Найдите значение выражения:

Решение:

Задание 20. Найдите значение выражения:

Решение:

Задание 21. Найдите значение выражения:

Решение:

Задание 22. Найдите значение выражения:

Решение:

Задание 23. Найдите значение выражения:

Решение:

Задание 24. Найдите значение выражения:

Решение:

Задание 25. Найдите значение выражения:

Решение:

Задание 26. Найдите значение выражения:

Решение:

Задание 27. Найдите значение выражения:

Решение:

Задание 28. Найдите значение выражения:

Решение:

Задание 29. Найдите значение выражения:

Решение:

Задание 30. Найдите значение выражения:

Решение:

Задание 31. Найдите значение выражения:

Решение:

Задание 32. Найдите значение выражения:

Решение:

Задание 33. Найдите значение выражения:

Решение:

Задание 34. Вынести множитель из-под знака корня:

Решение:

Задание 35. Вынести множитель из-под знака корня:

Решение:

Задание 36. Вынести множитель из-под знака корня:

Решение:

Задание 37. Вынести множитель из-под знака корня:

Решение:

Задание 38. Вынести множитель из-под знака корня:

Решение:

Задание 39. Вынести множитель из-под знака корня:

Решение:

Задание 40. Вынести множитель из-под знака корня:

Решение:

Задание 41. Вынести множитель из-под знака корня:

Решение:

Задание 42. Вынести множитель из-под знака корня:

Решение:

Задание 43. Вынести множитель из-под знака корня:

Решение:

Задание 44. Вынести множитель из-под знака корня в следующих выражениях:

Решение:

Задание 45. Внести множитель под знак корня:

Решение:

Задание 46. Внести множитель под знак корня:

Решение:

Задание 47. Внести множитель под знак корня:

Решение:

Задание 48. Внести множитель под знак корня:

Решение:

Задание 49. Внести множитель под знак корня:

Решение:

Задание 50. Внести множитель под знак корня в следующих выражениях:

Решение:

Задание 51. Упростить выражение:

Решение:

Задание 52. Упростить выражение:

Решение:

Задание 53. Упростить выражение:

Решение:

Задание 54. Упростить выражение:

Решение:

Задание 55. Упростить выражение:

Решение:

Задание 56. Упростить выражение:

Решение:

Задание 57. Упростить выражение:

Решение:

Задание 58. Упростить выражение:

Решение:

Задание 59. Упростить выражение:

Решение:

Задание 60. Упростить выражение:

Решение:


Понравился урок?
Вступай в нашу новую группу Вконтакте и начни получать уведомления о новых уроках



Возникло желание поддержать проект?
Используй кнопку ниже

Навигация по записям

Решение радикальных уравнений — ChiliMath

Чтобы научиться решать радикальные уравнения, требуется много практики и знакомство с различными типами задач. Цель этого урока — показать вам детально проработанные решения некоторых проблем с разным уровнем сложности.

Что такое радикальное уравнение?

Уравнение, в котором переменная содержится внутри символа корня или имеет рациональный показатель степени. В частности, мы будем иметь дело с квадратным корнем, который является следствием того, что показатель степени равен \ Large {1 \ over 2}.

Ключевые шаги:

1) Изолировать радикальный символ на одной стороне уравнения

2) Возведите обе части уравнения в квадрат, чтобы исключить радикальный символ

.

3) Решите уравнение, которое появляется после возведения в квадрат

4) Сравните свои ответы с исходным уравнением, чтобы избежать посторонних значений


Примеры решения радикальных уравнений

Пример 1 : Решите радикальное уравнение

Радикал сам по себе находится на одной стороне, поэтому можно возвести обе части уравнения в квадрат, чтобы избавиться от символа радикала.Затем выполните обычные шаги для решения линейных уравнений.

Вы должны ВСЕГДА проверять свои ответы, чтобы убедиться, что они «действительно» решения. Некоторые ответы из ваших расчетов могут быть посторонними. Подставим x = 16 обратно в исходное радикальное уравнение, чтобы проверить, дает ли оно истинное утверждение.

Да, проверяет, поэтому x = 16 — решение.


Пример 2 : Решите радикальное уравнение

Установка выглядит хорошо, потому что радикал снова изолирован с одной стороны.Таким образом, я могу возвести обе стороны в квадрат, чтобы удалить этот квадратный корень. Будьте осторожны с правой стороной при возведении двучлена (x − 1) в квадрат. Вы должны правильно применить метод FOIL.

Мы перемещаем все члены в правую часть уравнения и затем выносим трехчлен за скобки. Применяя свойство нулевого произведения, мы получаем значения x = 1 и x = 3 .

Внимание : Всегда проверяйте рассчитанные вами значения из исходного радикального уравнения, чтобы убедиться, что они являются истинными ответами, а не посторонними или «ложными».

Выглядит хорошо для обоих решенных значений x после проверки, поэтому наши решения равны x = 1 и x = 3 .


Пример 3 : Решите радикальное уравнение

Нам нужно признать, что радикальный символ еще не изолирован с левой стороны. Это означает, что нам нужно избавиться от этого −1 , прежде чем возвести в квадрат обе части уравнения. Простой шаг добавления обеих сторон на 1 должен решить эту проблему. После этого «новое» уравнение похоже на те, которые мы рассмотрели до сих пор.

Наши возможные решения: x = −2 и x = 5 . Обратите внимание, я использую слово «возможно», потому что оно не является окончательным, пока мы не выполним наш процесс проверки, сравнивая наши значения с исходным радикальным уравнением.

Поскольку мы приходим к ложному утверждению, когда x = −2, это значение x считается посторонним , поэтому мы его игнорируем! Остается один верный ответ: x = 5 .


Пример 4 : Решите радикальное уравнение

Левая сторона выглядит немного неаккуратно из-за двух радикальных символов.Но не все так плохо! Всегда помните ключевые шаги, предложенные выше. Поскольку оба квадратных корня находятся на одной стороне, это означает, что оно определенно готово для возведения в квадрат всего радикального уравнения.

Итак, для нашего первого шага давайте возведем обе стороны в квадрат и посмотрим, что произойдет.

Для этого типа задач совершенно нормально увидеть другой радикальный символ после первого применения возведения в квадрат. Хорошие новости заключаются в том, что остался только один. С этого момента попробуйте снова изолировать единственный радикал на левой стороне, что должно заставить нас переместить остальные на противоположную сторону.

Как видите, это упрощенное радикальное уравнение , безусловно, знакомое . Используйте обычный способ решения и убедитесь, что вы всегда сверяете решенные значения x с исходным радикальным уравнением.

Я оставлю вам проверить, действительно ли x = 4 — это решение.


Пример 5 : Решите радикальное уравнение

Эта задача очень похожа на пример 4. Единственная разница в том, что на этот раз оба радикала имеют биномиальные выражения.Подход также состоит в том, чтобы выровнять обе стороны, так как радикалы находятся на одной стороне, и упростить. Но нам нужно выполнить второе применение возведения в квадрат, чтобы полностью избавиться от символа квадратного корня.

Решение: x = 2 . Вы можете проверить это, подставив значение обратно в исходное радикальное уравнение и убедившись, что оно дает истинное утверждение.


Пример 6 : Решите радикальное уравнение

Похоже, наш первый шаг — возвести обе стороны в квадрат и посмотреть, что получится потом.Не забывайте комбинировать одинаковые термины каждый раз, когда возводите стороны в квадрат. Если случится так, что после первого применения процесса возведения в квадрат образуется еще один радикальный символ, имеет смысл сделать это еще раз. Помните, наша цель — избавиться от радикальных символов, чтобы освободить переменную, которую мы пытаемся решить или изолировать.

Что ж, похоже, нам нужно будет снова возвести обе стороны в квадрат из-за нового сгенерированного радикального символа. Но мы должны сначала изолировать радикал на одной стороне уравнения, прежде чем делать это.Я сохраню квадратный корень слева, и это заставит меня сдвинуть все вправо.

Пока все в порядке! Теперь пришло время снова возвести обе стороны в квадрат, чтобы окончательно устранить радикал.

Будьте осторожны, возводя в квадрат левую часть уравнения. Вы также должны возвести в квадрат , что −2 слева от радикала.

Теперь у нас есть квадратное уравнение в стандартной форме. Лучший способ найти x — использовать квадратичную формулу, где a = 7, b = 8 и c = −44.

Итак, возможные решения: x = 2 и x = {{- 22} \ over 7}.

Я оставлю вам проверить эти два значения «x» обратно в исходное радикальное уравнение. Надеюсь, вы согласны с тем, что x = 2 — единственное решение, а другое значение — постороннее решение, поэтому не обращайте на него внимания!


Пример 7 : Решите радикальное уравнение

Есть два способа решить эту проблему. Я мог немедленно возвести обе стороны в квадрат, чтобы избавиться от радикалов, или сначала умножить два радикала, а затем возвести в квадрат.При правильном выполнении обе процедуры должны дать одинаковые ответы. Для этого я воспользуюсь вторым подходом.

Затем переместите все в левую сторону и решите полученное квадратное уравнение. Вы можете использовать квадратичную формулу для ее решения, но, поскольку ее легко факторизовать, я просто вычеркну ее.

Возможные решения: x = {{- 5} \ over 2} и x = 3.

Я оставлю это вам, чтобы проверить ответы. Единственный ответ должен быть x = 3, что делает другое решение лишним.


Рабочие листы по радикальным уравнениям


Возможно, вас заинтересует:

Упрощение радикальных выражений
Сложение и вычитание радикальных выражений
Умножение радикальных выражений
Рационализация знаменателя

упрощающих радикалов

упрощающих радикалов
Чтобы упростить радикалы, вместо того, чтобы искать точные квадраты или идеальные кубы внутри числа или переменной, как это показано в большинстве книг, я решаю задачи по-другому, и вот как.

Вот шаги, необходимые для упрощения радикалов:

Шаг 1 : Найдите разложение числа внутри корня на простые множители. Начните с деления числа на первое простое число 2 и продолжайте деление на 2, пока не получите десятичную дробь или остаток. Затем разделите на 3, 5, 7 и т. Д., Пока не останутся только простые числа. Щелкните ссылку, чтобы увидеть несколько примеров первичной факторизации. Также учитывайте любые переменные внутри радикала.
Шаг 2 : Определите индекс радикала. Индекс сообщает вам, сколько чисел вам нужно собрать, чтобы иметь возможность переместить это число или переменную из радикала во вне радикала. Например, если индекс равен 2 (квадратный корень), то вам нужно два одинаковых вида, чтобы переместиться изнутри радикала во вне радикала. Если индекс равен 3 (кубический корень), то вам нужны тройки одного сорта, чтобы переместиться из корня в корень вне корня.
Шаг 3 : Переместите каждую группу чисел или переменных из радикала в радикал. Если чисел или переменных недостаточно, чтобы создать группу из двух, трех или чего-то еще, оставьте эти числа или переменные внутри радикала. Обратите внимание, что каждая группа чисел или переменных записывается один раз, когда они выходят за пределы радикала, потому что теперь они составляют одну группу.
Шаг 4 : Упростите выражения как внутри, так и вне корня путем умножения.Умножьте все числа и переменные внутри радикала вместе. Умножьте все числа и переменные вне корня вместе.

Пример 1 — Упростить:

Шаг 1 : Найдите разложение числа внутри корня на простые множители.
Шаг 2 : Определите индекс корня. В этом случае индекс равен двум, потому что это квадратный корень, а это значит, что нам нужны два одинаковых типа.
Шаг 3 : Переместите каждую группу чисел или переменных из радикала во вне радикала. В этом случае пара двойки и тройки вышла за пределы радикала.
Шаг 4 : Упростите выражения как внутри, так и вне корня путем умножения.

Пример 2 — Упростить:

Шаг 1 : Найдите разложение числа внутри корня на простые множители и разложите на множители каждую переменную внутри корня.
Шаг 2 : Определите индекс корня. В этом случае индекс равен трем, потому что это кубический корень, а это значит, что нам нужны три одинаковых.
Шаг 3 : Переместите каждую группу чисел или переменных из радикала во вне радикала. В этом случае тройки, x (две группы) и y вышли за пределы радикала.
Шаг 4 : Упростите выражения как внутри, так и вне корня путем умножения.

Щелкните здесь для практических задач

Пример 3 — Упростить:

Шаг 1 : Найдите разложение числа внутри корня на простые множители и разложите на множители каждую переменную внутри корня.
Шаг 2 : Определите индекс корня. В данном случае индекс равен пяти, потому что это корень пятой степени, а это значит, что нам нужны пять одинаковых чисел.
Шаг 3 : Переместите каждую группу чисел или переменных из радикала во вне радикала. В этом случае тройки, x и y выходят за пределы радикала.
Шаг 4 : Упростите выражения как внутри, так и вне корня путем умножения.

Щелкните здесь для практических задач

Пример 4 — Упростить:

Шаг 1 : Найдите разложение числа внутри корня на простые множители и разложите на множители каждую переменную внутри корня.
Шаг 2 : Определите индекс корня. В этом случае индекс равен двум, потому что это квадратный корень, а это значит, что нам нужны два одинаковых типа.
Шаг 3 : Переместите каждую группу чисел или переменных из радикала во вне радикала. В этом случае двойки, тройки, x (две группы) и y (четыре группы) вышли за пределы радикала.
Шаг 4 : Упростите выражения как внутри, так и вне корня путем умножения.

Щелкните здесь для практических задач

Пример 5 — Упростить:

Шаг 1 : Найдите разложение числа внутри корня на простые множители и разложите на множители каждую переменную внутри корня.
Шаг 2 : Определите индекс корня. В данном случае индекс равен четырем, потому что это корень четвертой степени, а это значит, что нам нужны четыре одинаковых числа.
Шаг 3 : Переместите каждую группу чисел или переменных из радикала во вне радикала. В этом случае пара двоек и y вышла за пределы радикала.
Шаг 4 : Упростите выражения как внутри, так и вне корня путем умножения.

Щелкните здесь для практических задач

Пример 6 — Упростить:

Шаг 1 : Найдите разложение числа внутри корня на простые множители и разложите на множители каждую переменную внутри корня.
Шаг 2 : Определите индекс корня. В этом случае индекс равен трем, потому что это кубический корень, а это значит, что нам нужны три одинаковых.
Шаг 3 : Переместите каждую группу чисел или переменных из радикала во вне радикала. В этом случае двойки, тройки, х и у (две группы) вышли за пределы радикала.
Шаг 4 : Упростите выражения как внутри, так и вне корня путем умножения.

Щелкните здесь для практических задач

Решение радикальных уравнений | Начальная алгебра

Цели обучения

  • Решите радикальные уравнения
    • Выделить квадратные корни в уравнениях и найти переменную
    • Определите посторонние решения радикальных уравнений
  • Квадратные корни и завершение квадрата для решения радикальных уравнений
    • Использование квадратных корней для решения квадратных уравнений
    • Заполните квадрат, чтобы решить квадратное уравнение
  • Использование квадратичной формулы для решения квадратных уравнений
    • Напишите квадратное уравнение в стандартной форме и определите значения a , b и c в квадратном уравнении стандартной формы.
    • Используйте квадратичную формулу, чтобы найти все реальные решения.
    • Решите прикладные задачи, требующие использования квадратичной формулы.

Основная стратегия решения радикальных уравнений состоит в том, чтобы сначала выделить радикальный член, а затем возвести обе части уравнения в степень, чтобы удалить радикал. (Причина использования степеней станет ясна через мгновение.) Это тот же тип стратегии, который вы использовали для решения других, нерадикальных уравнений — измените выражение, чтобы выделить переменную, которую вы хотите знать, а затем решите полученное уравнение .{2}} = х [/ латекс]. (Это свойство позволяет вам «удалить» радикалы из ваших уравнений.)

Начнем с радикального уравнения, которое можно решить за несколько шагов: [latex] \ sqrt {x} -3 = 5 [/ latex].

Пример

Решить. [латекс] \ sqrt {x} -3 = 5 [/ латекс]

Показать решение
Добавьте 3 к обеим сторонам, чтобы изолировать переменный член в левой части уравнения.

[латекс] \ begin {array} {r} \ sqrt {x} -3 \, \, \, = \, \, \, 5 \\\ подчеркивание {+3 \, \, \, \, \, \, \, + 3} \ end {array} [/ latex]

Соберите похожие термины.{2}} \\ x = 64 \ end {array} [/ latex]

Ответ

[латекс] x = 64 [/ latex] — это решение для [latex] \ sqrt {x} -3 = 5 [/ latex]

Чтобы проверить свое решение, вы можете заменить 64 вместо x в исходном уравнении. [Латекс] \ sqrt {64} -3 = 5 [/ латекс]? Да, квадратный корень из 64 равен 8, а [латекс] 8−3 = 5 [/ latex].

Обратите внимание, как вы объединили одинаковые члены, а затем возводили в квадрат обе стороны уравнения в этой задаче. Это стандартный метод удаления радикала из уравнения.Важно выделить радикал на одной стороне уравнения и максимально упростить перед возведением в квадрат . Чем меньше членов перед возведением в квадрат, тем меньше дополнительных членов будет сгенерировано в процессе возведения в квадрат.

В приведенном выше примере только переменная x находилась под радикалом. Иногда вам нужно будет решить уравнение, которое содержит несколько членов под радикалом. Выполните те же шаги, чтобы решить их, но обратите внимание на критическую точку — возведите в квадрат обе стороны уравнения, а не отдельные члена .{2}} = x + 8 [/ латекс]. Теперь упростите уравнение и решите относительно x .

[латекс] \ begin {array} {r} x + 8 = 9 \\ x = 1 \ end {array} [/ latex]

Проверьте свой ответ. Подстановка 1 вместо x в исходном уравнении дает истинное утверждение, поэтому решение является правильным.

[латекс] \ begin {массив} {r} \ sqrt {1 + 8} = 3 \\\ sqrt {9} = 3 \\ 3 = 3 \ end {array} [/ latex]

Ответ

[латекс] x = 1 [/ latex] — это решение [latex] \ sqrt {x + 8} = 3 [/ latex].

В следующем видео мы покажем, как решать простые радикальные уравнения.{2}} \, \, \, \ end {array} [/ latex]

Упростите уравнение и решите относительно x .

[латекс] \ begin {array} {r} 2x + 3 = 25 \\ 2x = 22 \ x = 11 \ end {array} [/ latex]

Проверьте свой ответ. Подстановка 11 вместо x в исходном уравнении дает истинное утверждение, поэтому решение является правильным.

[латекс] \ begin {array} {r} 1+ \ sqrt {2 (11) +3} = 6 \\ 1+ \ sqrt {22 + 3} = 6 \\ 1+ \ sqrt {25} = 6 \\ 1 + 5 = 6 \\ 6 = 6 \ end {array} [/ latex]

Ответ

[латекс] x = 11 [/ latex] — это решение для [latex] 1+ \ sqrt {2x + 3} = 6 [/ latex].{2} [/ латекс].

  • Как только радикал будет удален, решите неизвестное.
  • Отметьте все ответы.
  • Определите посторонние решения

    Следование правилам важно, но не менее важно уделять внимание математике, стоящей перед вами, особенно при решении радикальных уравнений. Взгляните на следующую задачу, которая демонстрирует потенциальную ловушку квадратуры обеих сторон для удаления радикала.

    Пример

    Решить. [латекс] \ sqrt {a-5} = — 2 [/ латекс]

    Показать решение
    Выровняйте обе стороны, чтобы удалить термин [латекс] a – 5 [/ латекс] из радикала.{2}} [/ латекс]

    Напишите упрощенное уравнение и решите относительно a .

    [латекс] \ begin {array} {r} a-5 = 4 \\ a = 9 \ end {array} [/ latex]

    Теперь проверьте решение, подставив [latex] a = 9 [/ latex] в исходное уравнение.

    Это не проверяет!

    [латекс] \ begin {array} {r} \ sqrt {9-5} = — 2 \\\ sqrt {4} = — 2 \\ 2 \ ne -2 \ end {array} [/ latex]

    Ответ

    Нет решения.

    Посмотрите на это — ответ [latex] a = 9 [/ latex] не дает истинного утверждения, если подставить его обратно в исходное уравнение.Что случилось?

    Проверьте исходную проблему: [latex] \ sqrt {a-5} = — 2 [/ latex]. Обратите внимание, что радикал установлен равным [латекс] -2 [/ latex], и вспомните, что главный квадратный корень из числа может быть только положительным . Это означает, что никакое значение для a не приведет к радикальному выражению, положительный квадратный корень которого равен [latex] −2 [/ latex]! Вы могли заметить это сразу и сделать вывод, что для и решений не существует.

    Неправильные значения переменной, например, введенные в результате процесса возведения в квадрат, называются посторонними решениями .Посторонние решения могут выглядеть как реальное решение, но вы можете идентифицировать их, потому что они не создадут истинного утверждения, когда будут подставлены обратно в исходное уравнение. Это одна из причин, почему проверка вашей работы так важна — если вы не проверяете свои ответы, подставляя их обратно в исходное уравнение, вы можете вводить в проблему посторонние решения.
    В следующем видео-примере мы решаем более радикальные уравнения, которые могут иметь посторонние решения.

    Обратите внимание на следующую проблему.{2}} + 7x + 6 = 0 \, \, \, \, \, \, \, \, \, \, \, \, \, \, \\\ влево (x + 6 \ вправо) \ left (x + 1 \ right) = 0 \, \, \, \, \, \, \, \, \, \, \, \, \, \, \ end {array} [/ latex]

    Установите каждый коэффициент равным нулю и решите относительно x.

    [латекс] \ begin {array} {c} \ left (x + 6 \ right) = 0 \, \, \ text {или} \, \, \ left (x + 1 \ right) = 0 \\ x = -6 \ text {или} x = -1 \ end {array} [/ latex]

    Теперь проверьте оба решения, подставив их в исходное уравнение.

    Поскольку [latex] x = −6 [/ latex] дает ложное утверждение, это постороннее решение.

    [латекс] \ begin {array} {l} -6 + 4 = \ sqrt {-6 + 10} \\\, \, \, \, \, \, \, \, — 2 = \ sqrt {4 } \\\, \, \, \, \, \, \, \, — 2 = 2 \\\ text {FALSE!} \\\\\\ — 1 + 4 = \ sqrt {-1 + 10} \\\, \, \, \, \, \, \, \, \, \, \, \, \, \, \, 3 = \ sqrt {9} \\\, \, \, \, \ , \, \, \, \, \, \, \, \, \, \, 3 = 3 \\\ текст {ИСТИНА!} \ End {array} [/ latex]

    Ответ

    [латекс] x = -1 [/ латекс] — единственное решение

    Пример

    Решить. [латекс] 4+ \ sqrt {x + 2} = x [/ латекс]

    Показать решение
    Выделите радикальный термин.

    [латекс] \ sqrt {x + 2} = x-4 [/ латекс]

    Квадрат с обеих сторон, чтобы удалить член [латекс] x + 2 [/ латекс] из корня.{2}} — 9x + 14 \\\, \, \, \, \, \, \, \, \, \, \, 0 = \ left (x-7 \ right) \ left (x-2 \ справа) \ end {array} [/ latex]

    Установите каждый коэффициент равным нулю и решите относительно x.

    [латекс] \ begin {array} {c} \ left (x-7 \ right) = 0 \ text {or} \ left (x-2 \ right) = 0 \\ x = 7 \ text {or} x = 2 \ end {array} [/ latex] [latex] [/ latex]

    Теперь проверьте оба решения, подставив их в исходное уравнение.

    Поскольку [latex] x = 2 [/ latex] дает ложное утверждение, это постороннее решение.

    [латекс] \ begin {array} {r} 4+ \ sqrt {7 + 2} = 7 \\ 4+ \ sqrt {9} = 7 \\ 4 + 3 = 7 \ 7 = 7 \ text {ИСТИНА!} \\\\ 4+ \ sqrt {2 + 2} = 2 \\ 4+ \ sqrt {4} = 2 \\ 4 + 2 = 2 \\ 6 = 2 \\\ текст {ЛОЖЬ!} \ end {array} [/ latex]

    Ответ

    [латекс] х = 7 [/ латекс] — единственное решение.{2} = 9 [/ latex], вы ищете всех чисел , квадрат которых равен 9. Для [latex] \ sqrt {9} [/ latex] вам нужен только основной (неотрицательный) квадратный корень. Отрицательное значение главного квадратного корня равно [латекс] — \ sqrt {9} [/ latex]; оба будут [латекс] \ pm \ sqrt {9} [/ latex]. Если перед радикальным знаком нет символа, требуется только неотрицательное значение!

    В приведенном выше примере вы можете легко извлечь квадратный корень из обеих частей, потому что на каждой стороне есть только один член.{2}} = 8 \\\, \, \, x = \ pm \ sqrt {8} \\\, \, \, \, \, \, = \ pm \ sqrt {(4) (2)} \\\, \, \, \, \, \, = \ pm \ sqrt {4} \ sqrt {2} \\\, \, \, \, \, \, = \ pm 2 \ sqrt {2} \ end {array} [/ latex]

    Ответ

    [латекс] x = \ pm 2 \ sqrt {2} [/ латекс]

    В следующем видео мы покажем больше примеров решения простых квадратных уравнений с использованием квадратных корней.

    Иногда возводится в квадрат больше, чем x :

    Пример

    Решить. {2} –50 = 0 [/ latex]

    Показать решение
    Опять же, извлечение квадратного корня из обеих частей на этом этапе оставит то, с чем вы не сможете работать слева.{2} = 50 \, \, \, \, \, \, \, \, \, \, \\ x-2 = \ pm \ sqrt {50} \ end {array} [/ latex]

    Чтобы выделить x слева, вам нужно добавить 2 с обеих сторон.

    Обязательно упростите радикал, если это возможно.

    [латекс] \ begin {array} {l} x = 2 \ pm \ sqrt {50} \\\, \, \, \, = 2 \ pm \ sqrt {(25) (2)} \\\, \, \, \, = 2 \ pm \ sqrt {25} \ sqrt {2} \\\, \, \, \, = 14 \ pm 5 \ sqrt {2} \ end {array} [/ latex]

    Ответ

    [латекс] x = 2 \ pm 5 \ sqrt {2} [/ латекс]

    В этом видео-примере вы увидите больше примеров решения квадратных уравнений с использованием квадратных корней.{2} = 8 [/ латекс]

    Теперь вы можете использовать свойство квадратного корня. Чтобы изолировать x , необходимы некоторые дополнительные действия.

    [латекс] \ begin {array} {r} 2x + 5 = \ pm \ sqrt {8} \, \, \, \, \, \, \, \, \, \, \, \, \, \ , \, \, \\\\ 2x = -5 \ pm \ sqrt {8} \, \, \, \, \, \\\\ x = — \ frac {5} {2} \ pm \ frac { 1} {2} \ sqrt {8} \ end {array} [/ latex]

    По возможности упрощайте радикал.

    [латекс] \ begin {array} {l} x = — \ frac {5} {2} \ pm \ frac {1} {2} \ sqrt {4} \ sqrt {2} \\\\ x = — \ frac {5} {2} \ pm \ frac {1} {2} (2) \ sqrt {2} \\\\ x = — \ frac {5} {2} \ pm \ sqrt {2} \ end {array} [/ latex]

    Ответ

    [латекс] x = — \ frac {5} {2} \ pm \ sqrt {2} [/ latex]

    Один из способов решить квадратные уравнения — это заполнить квадрат .{2} + bx [/ латекс].

    В этом примере площадь всего прямоугольника задается как [латекс] x \ left (x + b \ right) [/ latex].

    Теперь давайте превратим этот прямоугольник в квадрат. Сначала разделите красный прямоугольник с площадью bx на два равных прямоугольника, каждый с площадью [латекс] \ frac {b} {2} x [/ latex]. Затем поверните и переместите один из них. Вы не изменили размер красной области — она ​​все равно составляет [latex] bx [/ latex].

    Красные прямоугольники теперь составляют две стороны квадрата, показанного белым.{2}} [/ latex] всегда положительно, так как это квадрат числа. Когда вы завершаете квадрат, вы всегда добавляете положительное значение.

    В следующем видео мы покажем больше примеров того, как найти постоянные члены, которые сделают трехчлен совершенным квадратом.

    Вы можете использовать завершение квадрата, чтобы решить квадратное уравнение, которое нельзя решить с помощью факторизации.

    Давайте начнем с того, что посмотрим, что происходит, когда вы заполняете квадрат в уравнении.{2} = 40 [/ латекс]

    Используйте свойство квадратного корня. Не забудьте включить как положительный, так и отрицательный квадратный корень, иначе вы пропустите одно из решений.

    [латекс] x-6 = \ pm \ sqrt {40} [/ латекс]

    Решите относительно x , прибавив 6 к обеим сторонам. При необходимости упростите.

    [латекс] \ begin {array} {l} x = 6 \ pm \ sqrt {40} \\\, \, \, \, = 6 \ pm \ sqrt {4} \ sqrt {10} \\\, \, \, \, = 18 \ pm 2 \ sqrt {10} \ end {array} [/ latex]

    Ответ

    [латекс] x = 6 \ pm 2 \ sqrt {10} [/ латекс]

    В этом последнем видео мы решаем больше квадратных уравнений, завершая квадрат.{2} = 0 [/ латекс]

    Извлеките квадратные корни из обеих сторон. Обычно требуются как положительные, так и отрицательные квадратные корни, но 0 не является ни положительным, ни отрицательным. 0 имеет только один корень.

    [латекс] x + 8 = 0 [/ латекс]

    Решите для x.

    [латекс] x = -8 [/ латекс]

    Ответ

    [латекс] x = -8 [/ латекс]

    Присмотритесь к этой проблеме поближе, и вы увидите что-то знакомое. Вместо того, чтобы строить квадрат, попробуйте прибавить 47 к обеим сторонам уравнения.{2} + 16x + 64 = 0 [/ латекс]. Можете ли вы разложить это уравнение на множители, используя группировку? (Подумайте о двух числах, произведение которых равно 64, а сумма равна 16).

    Конечно, это может быть учтено как [латекс] (x + 8) (x + 8) = 0 [/ latex]! Очень полезно знать, как заполнить квадрат, но это не всегда единственный способ решить уравнение.

    Используйте квадратичную формулу для решения квадратных уравнений

    Квадратичная формула

    Вы можете решить любое квадратное уравнение, заполнив квадрат — записав часть уравнения в виде трехчлена полного квадрата.{2} + bx + c = 0 [/ латекс]. Чтобы использовать его, выполните следующие действия.

    • Сначала поместите уравнение в стандартную форму.
    • Определите коэффициенты, a , b, и c. Будьте осторожны, добавляя отрицательные знаки, если вычитаются члены bx или c .
    • Подставьте значения коэффициентов в формулу корней квадратного уравнения.
    • Максимально упрощайте.
    • Используйте [латекс] \ pm [/ latex] перед радикалом, чтобы разделить решение на два значения: одно, в котором складывается квадратный корень, и второе, в котором он вычитается .{2}} — 4 (1) (- 5)}} {2 (1)} \ end {array} [/ latex]

      Упростите, стараясь получить правильные знаки.

      [латекс] x = \ frac {-4 \ pm \ sqrt {16 + 20}} {2} [/ латекс]

      Еще немного упростите.

      [латекс] x = \ frac {-4 \ pm \ sqrt {36}} {2} [/ латекс]

      Упростите радикал: [латекс] \ sqrt {36} = 6 [/ латекс].

      [латекс] x = \ frac {-4 \ pm 6} {2} [/ латекс]

      Разделите и упростите, чтобы найти решения квадратного уравнения. Обратите внимание, что в одном случае добавляется 6, а в другом — вычитается.{2} +4 \ влево (-5 \ вправо) = 5 \, \, \, \, \, \\ 25-20 = 5 \, \, \, \, \, \\ 5 = 5 \, \ , \, \, \, \ end {array} [/ latex]

      Вы получаете два истинных утверждения, так что вы знаете, что оба решения работают: [latex] x = 1 [/ latex] или [latex] -5 [/ latex]. Вы успешно решили уравнение с помощью квадратной формулы!

      Сила квадратичной формулы в том, что с ее помощью можно решить любое квадратное уравнение , даже те, в которых поиск комбинаций чисел не сработает.

      В следующем видео мы показываем пример использования формулы корней квадратного уравнения для решения уравнения с двумя действительными решениями.{2} -2 \ left (4 \ right) = 6 \ left (4 \ right) -16 \\ 16-8 = 24-16 \, \, \, \, \, \, \\ 8 = 8 \ , \, \, \, \, \, \, \, \, \, \, \, \, \, \, \, \, \, \, \, \, \, \ end {array} [/ латекс]

      В следующем видео мы показываем пример использования квадратной формулы для решения квадратного уравнения, имеющего одно повторяющееся решение.

      В этом видео-примере мы показываем, что решения квадратных уравнений могут иметь рациональные ответы.

      Применение квадратичной формулы

      Квадратные уравнения широко используются в науке, бизнесе и технике.Квадратные уравнения обычно используются в ситуациях, когда две вещи перемножаются и оба зависят от одной и той же переменной. Например, при работе с площадью, если оба измерения записаны в терминах одной и той же переменной, вы используете квадратное уравнение. Поскольку количество проданного продукта часто зависит от цены, вы иногда используете квадратное уравнение, чтобы представить доход как произведение цены и проданного количества. Квадратные уравнения также используются, когда задействована сила тяжести, например, путь шара или форма тросов в подвесном мосту.

      Очень распространенное и простое для понимания приложение — это высота мяча, брошенного в землю над зданием. Поскольку гравитация заставляет мяч ускоряться при падении, квадратное уравнение можно использовать для оценки его высоты в любое время до того, как он упадет на землю. Примечание. Уравнение не совсем точное, потому что трение с воздуха немного замедлит мяч. Для наших целей это достаточно близко.

      Пример

      Мяч брошен зданием с высоты 200 футов над землей.{2}} — 4 (-16) (200)}} {2 (-16)} [/ латекс]

      Упростить. Будьте очень осторожны со знаками.

      [латекс] \ begin {array} {l} t = \ frac {10 \ pm \ sqrt {100 + 12800}} {- 32} \\\, \, = \ frac {10 \ pm \ sqrt {12900} } {- 32} \ end {array} [/ latex]

      Используйте калькулятор, чтобы найти оба корня.

      t составляет приблизительно [латекс] -3,86 [/ латекс] или [латекс] 3,24 [/ латекс].

      Рассмотрим корни логически. Одно решение, [латекс] -3,86 [/ латекс], не может быть временем, потому что это отрицательное число. Другое решение, [латекс] 3.24 [/ latex] секунды должно быть, когда мяч ударяется о землю.

      Ответ

      Мяч падает о землю примерно [latex] через 3,24 [/ latex] секунды после броска.

      Приведенная ниже задача с областями не выглядит так, как будто она включает квадратичную формулу любого типа, и кажется, что проблема — это то, что вы уже много раз решали, просто умножая. Но чтобы ее решить, вам понадобится квадратное уравнение.

      Пример

      Боб сделал лоскутное одеяло размером 4 фута [латекс] \ раз [/ латекс] 5 футов.У него есть 10 квадратных футов ткани, которую он может использовать, чтобы добавить границу вокруг стеганого одеяла. Какую ширину он должен сделать каймой, чтобы использовать всю ткань? (Граница должна быть одинаковой ширины со всех четырех сторон.)

      Показать решение
      Набросайте проблему. Поскольку вы не знаете ширину границы, вы позволите переменной x представлять ширину.

      На схеме исходное лоскутное одеяло обозначено красным прямоугольником. Граница — это область между красной и синей линиями.

      Поскольку каждая сторона оригинального квилта 4 на 5 имеет границу шириной x , длина квилта с рамкой будет [латекс] 5 + 2x [/ латекс], а ширина будет [латекс] 4 + 2x [/ латекс].

      (Оба измерения записаны в терминах одной и той же переменной, и вы умножите их, чтобы получить площадь! Здесь вы можете начать думать, что квадратное уравнение может быть использовано для решения этой проблемы.)

      Вас интересует только район пограничных полос. Напишите выражение для области границы.

      Площадь границы = Площадь синего прямоугольника минус площадь красного прямоугольника

      Область границы [латекс] = \ left (4 + 2x \ right) \ left (5 + 2x \ right) — \ left (4 \ right) \ left (5 \ right) [/ latex]

      Для границы 10 квадратных футов ткани, поэтому установите область границы равной 10.{2}} — 4 (2) (- 5)}} {2 (2)} \ end {array} [/ latex]

      Упростить.

      [латекс] x = \ frac {-9 \ pm \ sqrt {121}} {4} = \ frac {-9 \ pm 11} {4} [/ latex]

      Найдите решения, убедившись, что [latex] \ pm [/ latex] оценивается для обоих значений.

      [латекс] \ begin {array} {c} x = \ frac {-9 + 11} {4} = \ frac {2} {4} = \ frac {1} {2} = 0,5 \\\\\ текст {или} \\\\ x = \ frac {-9-11} {4} = \ frac {-20} {4} = — 5 \ end {array} [/ latex]

      Игнорируйте решение [латекс] x = −5 [/ latex], так как ширина не может быть отрицательной.

      Ответ

      Ширина границы должна быть 0.5 футов

      В этом последнем видео мы показываем, как использовать формулу квадратичного уравнения для решения приложения, включающего рамку изображения.

      Сводка

      Распространенным методом решения радикальных уравнений является возведение обеих частей уравнения в любую степень, чтобы исключить знак радикала из уравнения. Но будьте осторожны — когда обе части уравнения возводятся в степень или даже , существует вероятность того, что будут введены посторонние решения.{2}} [/ latex] к обеим сторонам уравнения, чтобы сохранить равенство. Затем свойство квадратного корня можно использовать для нахождения x . При использовании свойства квадратного корня будьте осторожны, чтобы включить как главный квадратный корень, так и его противоположность. Обязательно упрощайте по мере необходимости. {2}} — 4ac}} {2a} [/ latex] находится путем завершения квадрата квадратного уравнения [латекс] [/ латекс].

      10.7: Решение радикальных уравнений — математика LibreTexts

      В этом разделе мы решим уравнения, которые содержат переменную в подкоренном выражении радикального выражения. Уравнение этого типа называется радикальным уравнением .

      Как обычно, при решении этих уравнений, то, что мы делаем с одной стороной уравнения, мы должны делать и с другой стороной. Как только мы изолировали радикал, наша стратегия будет заключаться в возведении обеих сторон уравнения в степень индекса.{п} = а \).

      Решите радикальное уравнение с одним радикалом

      1. Выделите радикал на одной стороне уравнения.
      2. Возведите обе части уравнения в степень индекса.
      3. Решите новое уравнение.
      4. Проверьте ответ в исходном уравнении.

      Знак корня указывает на главный или положительный корень. Если в уравнении есть радикал с четным индексом, равным отрицательному числу, это уравнение не будет иметь решения.

      Пример \ (\ PageIndex {2} \)

      Решить: \ (\ sqrt {9 k-2} + 1 = 0 \).

      Решение :

      Чтобы выделить радикал, вычтите \ (1 \) с обеих сторон.
      Упростить.
      Таблица 8.6.2

      Поскольку квадратный корень равен отрицательному числу, уравнение не имеет решения.

      Упражнение \ (\ PageIndex {3} \)

      Решение: \ (\ sqrt {2 r-3} + 5 = 0 \).

      Ответ

      нет решения

      Упражнение \ (\ PageIndex {4} \)

      Решить: \ (\ sqrt {7 s-3} + 2 = 0 \).

      Ответ

      нет решения

      Если одна сторона уравнения с квадратным корнем является биномом, мы используем образец произведения биномиальных квадратов, когда возводим его в квадрат.

      Определение \ (\ PageIndex {2} \)

      Биномиальные квадраты

      Не забудьте про средний срок!

      Пример \ (\ PageIndex {3} \)

      Решить: \ (\ sqrt {p-1} + 1 = p \).

      Решение :

      Решения: \ (p = 1, p = 2 \).

      Упражнение \ (\ PageIndex {5} \)

      Решение: \ (\ sqrt {x-2} + 2 = x \).

      Ответ

      \ (х = 2, х = 3 \)

      Упражнение \ (\ PageIndex {6} \)

      Решить: \ (\ sqrt {y-5} + 5 = y \).{3} \) Упростить. \ (5 х + 1 = -64 \) Решите уравнение. \ (5 х = -65 \) \ (х = -13 \) Проверьте ответ. Решение: \ (x = -13 \). Таблица 8.6,4

      Упражнение \ (\ PageIndex {7} \)

      Решить: \ (\ sqrt [3] {4 x-3} + 8 = 5 \)

      Ответ

      \ (х = -6 \)

      Упражнение \ (\ PageIndex {8} \)

      Решить: \ (\ sqrt [3] {6 x-10} + 1 = -3 \)

      Ответ

      \ (х = -9 \)

      Иногда уравнение может содержать рациональные показатели вместо радикала. Мы используем те же методы для решения уравнения, что и в случае радикала.{4} \) Упростить. \ (3 х-2 = 16 \) Решите уравнение. \ (3x = 18 \) \ (х = 6 \) Проверьте ответ. Решение: \ (x = 6 \). Таблица 8.{\ frac {1} {4}} + 5 = 7 \)

      Ответ

      \ (х = 6 \)

      Иногда решение радикального уравнения приводит к двум алгебраическим решениям, но одно из них может быть посторонним решением !

      Пример \ (\ PageIndex {6} \)

      Решение: \ (\ sqrt {r + 4} -r + 2 = 0 \).

      Решение :

      \ (\ sqrt {r + 4} -r + 2 = 0 \)
      Выделите радикал.{2} \)
      Упростите и решите уравнение.
      Если это квадратное уравнение, получим ноль с одной стороны.
      Разложите на множители правую сторону. \ (0 = г (г-5) \)
      Использовать свойство нулевого произведения. \ (0 = г \ квадр 0 = г-5 \)
      Решите уравнение. \ (г = 0 \ четырехугольник г = 5 \)
      Проверьте свой ответ.
      Решение: \ (r = 5 \).
      \ (r = 0 \) — экстремальное решение.
      Таблица 8.6.6

      Упражнение \ (\ PageIndex {11} \)

      Решить: \ (\ sqrt {m + 9} -m + 3 = 0 \)

      Ответ

      \ (м = 7 \)

      Упражнение \ (\ PageIndex {12} \)

      Решение: \ (\ sqrt {n + 1} -n + 1 = 0 \).

      Ответ

      \ (n = 3 \)

      Когда перед радикалом стоит коэффициент, мы также должны возвести его в степень индекса.

      Пример \ (\ PageIndex {7} \)

      Решить: \ (3 \ sqrt {3 x-5} -8 = 4 \).

      Решение :

      \ (3 \ sqrt {3 x-5} -8 = 4 \)
      Выделите радикальный член. \ (3 \ sqrt {3 x-5} = 12 \)
      Выделите радикал, разделив обе части на \ (3 \).{2} \)
      Упростите, а затем решите новое уравнение. \ (3 х-5 = 16 \)
      \ (3x = 21 \)
      Решите уравнение. \ (х = 7 \)
      Проверьте ответ.
      Решение: \ (x = 7 \).
      Таблица 8.6.7

      Упражнение \ (\ PageIndex {13} \)

      Решить: \ (2 \ sqrt {4 a + 4} -16 = 16 \).

      Ответ

      \ (а = 63 \)

      Упражнение \ (\ PageIndex {14} \)

      Решить: \ (3 \ sqrt {2 b + 3} -25 = 50 \)

      Ответ

      \ (b = 311 \)

      Решение радикальных уравнений: концепции | Purplemath

      Purplemath

      «Радикальное» уравнение — это уравнение, в котором по крайней мере одно выражение переменной заключено внутри радикала, обычно это квадратный корень.Большую часть этого урока мы будем работать с квадратными корнями.

      Например, это радикальное уравнение, потому что переменная находится внутри квадратного корня:

      Однако это уравнение не является радикальным:

      … потому что внутри радикала нет переменной.


      MathHelp.com

      Обычно мы решаем уравнения, выделяя переменную; то есть мы манипулируем уравнением, чтобы получить переменную с одной стороны от знака «равно» с числовым значением с другой стороны. Общий процесс изоляции в некотором смысле отменяет все, что было сделано с переменной в исходном уравнении.

      Например, предположим, что нам дано следующее линейное уравнение:

      Чтобы решить, мы должны отменить сложение 2; то есть мы бы вычли 2 из другой части уравнения:

      С другой стороны, учитывая уравнение –3 x = 12, мы бы решили, отменив умножение; то есть мы разделим на –3:

      Когда у нас есть переменная внутри квадратного корня, мы отменяем корень, делая противоположное; то есть мы возводим обе части уравнения в квадрат.Например, для уравнения

      мы бы решили возвести в квадрат обе части уравнения:

      Прежде чем мы погрузимся в отработанные примеры, нам нужно обсудить пару вопросов.


      Выпуск 1: Квадратные стороны, а не термины

      Решая уравнение, мы должны проделать то же самое с обеими сторонами уравнения; в частности, мы ничего не делаем с каждым членом в уравнении.В первом примере выше, решая « x + 2 = 5», я вычел 2 с каждой стороны, а не со всех трех членов.

      При решении радикального уравнения мы должны возвести обе стороны в квадрат ; мы никогда не должны пытаться уравнять каждый член. Например, я могу начать с истинного уравнения:

      … а затем возвести обе стороны в квадрат:

      Возводя в квадрат обе сторон уравнения, я получаю истинное уравнение.Вот как должна работать математика. Но если я попытаюсь возвести в квадрат слагаемых в левой части исходного утверждения выше, я не получу правильное значение.

      Очевидно, это неверно.

      В каждом случае я начал с верного утверждения. Когда я возложил в квадрат обе сторон этого истинного утверждения, я закончил истинным утверждением. Но когда я возложил в квадрат членов в левой части, я получил неверное утверждение: «25 = 49».

      Самая распространенная ошибка при решении радикальных уравнений — это возведение членов в квадрат. Всегда квадратные стороны, а не термины.


      Проблема 2. Проверьте свои ответы

      Мы всегда можем проверить наше решение уравнения, вставив это решение обратно в исходное уравнение и убедившись, что оно приводит к истинному утверждению. Например, в моем первом примере « x + 2 = 5» я получил решение x = 3.Я могу подтвердить это решение, подключив его обратно к исходному уравнению:

      Вы, вероятно, уже проверяли этот тип, когда впервые узнали о решении линейных уравнений. Но со временем вы отточили свои навыки и перестали проверять.

      Сложность решения радикальных уравнений состоит в том, что мы можем делать каждый шаг правильно, но все равно получать неправильный ответ. Это связано с тем, что сам акт сопоставления сторон может создать решений, которых никогда раньше не было.Например, я мог бы заявить следующее:

      Это, конечно, чушь. Но посмотрите, что происходит, когда я возведу в квадрат обе стороны:

      Я начал с того, что было , а не истинным, возложил в квадрат обе его стороны и закончил тем, что было истинным . Это не хорошо.

      Возведение обеих частей уравнения в квадрат — это «необратимый» шаг в том смысле, что, сделав шаг, мы не можем обязательно вернуться к тому, с чего начали.Возводя в квадрат, мы могли потерять часть исходной информации. (Это лишь одна из многих возможных ошибок в математике.)

      Чтобы увидеть, как это работает в нашем текущем контексте, давайте рассмотрим очень простое радикальное уравнение:

      Это уравнение не более верно, чем та чушь «–2 = 2», которую мы рассмотрели ранее, и это ерунда по той же причине: никакое положительное значение (в данном случае квадратный корень) не может равняться отрицательное число .

      Но предположим, что я не заметил, что это уравнение не может иметь никакого решения, и вместо этого бездумно продолжил возведение в квадрат обеих сторон:

      Возведя обе стороны в квадрат, я избавился от проблемного знака «минус», волшебным образом создав решение, которого раньше не существовало и которое фактически недействительно. Но я не обнаружу эту ошибку, если не забуду проверить свое решение! Вставляя значение моего решения в левую часть исходного уравнения, я проверяю, получаю ли я требуемое значение исходной правой части:

      Теперь я ясно вижу, что что-то не так.У меня не может быть отрицательного числа, равного положительному. Теперь я вижу, что реальный ответ для этого уравнения:

      .


      Есть другой способ взглянуть на эту трудность «без решения». Когда мы решаем уравнение, мы можем рассматривать процесс как попытку найти пересечение двух линий на графике. Левую часть уравнения можно изобразить как одну кривую, а правую часть уравнения можно изобразить как другую кривую. Решение исходного уравнения — это пересечение двух кривых.(Да, это означает, что вы можете использовать свой графический калькулятор, чтобы проверить свою работу.)

      Когда я решал « x + 2 = 5» выше, вы также могли сказать, что я пытался найти пересечение двух кривых:

      На графике показано, где пересекаются эти две линии:

      Точка пересечения находится на x = 3, что было значением решения, которое я нашел ранее.Точно так же, когда я решал уравнение

      , вы могли рассматривать это как попытку найти пересечение следующих двух кривых:

      График этих двух функций выглядит следующим образом:

      Как и раньше, решение равно x = 16.

      Но когда я пытался решить бессмысленное уравнение

      , я пытался найти пересечение графика радикальной функции y 1 = sqrt ( x ) и постоянной функции y 2 = –3, которые не пересекаются:


      Филиал


      Так что же произошло, когда я возложил в квадрат обе части этого бессмысленного уравнения? В некотором смысле, я как бы «возводил в квадрат» оба линейных уравнения и получил две новые строки:

      И, как показывает график, эти две новые линии на самом деле пересекаются!

      Как видите, возведение в квадрат обеих сторон исходного уравнения привело к решению, которому не место.И решение после возведения в квадрат , а не , работало в уравнении до возведения в квадрат, потому что исходные линий не пересекались. Это иллюстрирует, почему мне пришлось проверить свое решение, чтобы понять, что на самом деле ответ был «нет решения».

      Предупреждение: многие преподаватели не показывают много примеров (в классе или в домашнем задании) радикальных уравнений, решения которых на самом деле не работают. Но затем они проведут следующий тест по одному или нескольким из них. Вы должны ожидать, что будет радикальным уравнением «без решения» в тесте, поэтому не забудьте проверить свои решения.


      URL: https://www.purplemath.com/modules/solverad.htm

      Решение радикальных уравнений

      Радикальные уравнения

      Радикальное уравнение: Любое уравнение, которое содержит один или несколько радикалов с переменной в подкоренном выражении. — любое уравнение, которое содержит один или несколько радикалов с переменной в подкоренном выражении.Ниже приведены некоторые примеры радикальных уравнений, все из которых будут решены в этом разделе:

      Начнем с возведения в квадрат свойства равенства даных действительных чисел a и b , где a = b, затем a2 = b2 .; учитывая действительные числа a и b , мы имеем следующее:

      Другими словами, равенство сохраняется, если возвести обе части уравнения в квадрат.

      Обратное, с другой стороны, не обязательно верно:

      Это важно, потому что мы будем использовать это свойство для решения радикальных уравнений.Рассмотрим очень простое радикальное уравнение, которое можно решить путем осмотра:

      Здесь мы видим, что x = 9 — решение. Чтобы решить это уравнение алгебраически, воспользуйтесь свойством возведения в квадрат равенства и тем фактом, что (a) 2 = a2 = a, когда a положительно. Исключите квадратный корень, возведя в квадрат обе части уравнения следующим образом:

      В качестве проверки мы видим, что 9 = 3, как и ожидалось. Поскольку обратное к квадрату свойства равенства не обязательно верно, решения возведенного в квадрат уравнения могут не быть решениями исходного.Следовательно, возведение в квадрат обеих частей уравнения вводит возможность посторонних решений: решение, которое не решает исходное уравнение, или решения, которые не решают исходное уравнение. По этой причине мы должны проверять ответы, полученные при возведении в квадрат обеих частей уравнения.

      Пример 1: Решить: x − 1 = 5.

      Решение: Мы можем исключить квадратный корень, применив свойство возведения в квадрат равенства.

      Далее мы должны проверить.

      Ответ: Решение — 26.

      Пример 2: Решить: 5−4x = x.

      Решение: Начните с возведения в квадрат обеих частей уравнения.

      Осталось квадратное уравнение, которое можно решить с помощью факторизации.

      Поскольку вы соединили обе стороны в квадрат, вы должны проверить свои решения.

      После проверки вы можете увидеть, что x = −5 было посторонним; он не решал исходное радикальное уравнение. Не обращайте внимания на этот ответ. Это оставляет x = 1 в качестве единственного решения.

      Ответ: Решение x = 1.

      Обратите внимание на то, что в предыдущих двух примерах радикал изолирован с одной стороны уравнения. Обычно это не так. Шаги по решению радикальных уравнений, включающих квадратные корни, описаны в следующем примере.

      Пример 3: Решить: 2x − 5 + 4 = x.

      Решение:

      Шаг 1: Выделите квадратный корень. Начните с вычитания 4 из обеих частей уравнения.

      Шаг 2: Выровняйте обе стороны. Возведение в квадрат обеих сторон исключает квадратный корень.

      Шаг 3: Решите полученное уравнение. Здесь у вас остается квадратное уравнение, которое можно решить путем факторизации.

      Шаг 4: Проверьте решения в исходном уравнении. Квадрат обеих сторон вводит возможность посторонних решений; следовательно, требуется проверка.

      После проверки видим, что x = 3 — посторонний корень; он не решает исходное радикальное уравнение. Это оставляет x = 7 в качестве единственного решения.

      Ответ: Решение x = 7.

      Пример 4: Решить: 3x + 1−2x = 0.

      Решение: Начните с выделения термина с радикалом.

      Несмотря на то, что у члена слева есть коэффициент, он все равно считается изолированным. Напомним, что термины разделяются операторами сложения или вычитания.

      Решите полученное квадратное уравнение.

      Поскольку мы возведем обе стороны в квадрат, мы должны проверить наши решения.

      После проверки мы видим, что x = −34 было посторонним.

      Ответ: Решение — 3.

      Иногда оба возможных решения не имеют отношения к делу.

      Пример 5: Решить: 4−11x − x + 2 = 0.

      Решение: Начните с выделения радикала.

      Поскольку мы возведем обе стороны в квадрат, мы должны проверить наши решения.

      Поскольку оба возможных решения являются посторонними, уравнение не имеет решения.

      Ответ: Нет решения, Ø

      Свойство возведения в квадрат равенства распространяется на любую степень положительного целого числа n . Учитывая действительные числа a и b , мы имеем следующее:

      Это часто называют степенным свойством равенства Для любого положительного целого числа n и действительных чисел a и b , где a = b, затем an = bn .. Используйте это свойство вместе с тем фактом, что ( an) n = ann = a, когда a положительно, для решения радикальных уравнений с индексами больше 2.

      Пример 6: Решить: x2 + 43−2 = 0.

      Решение: Выделите радикал и затем возьмите обе части уравнения в куб.

      Проверить.

      Ответ: Решения — 2 и 2.

      Попробуй! Решить: 2x − 1 + 2 = x.

      Ответ: x = 5 (x = 1 — посторонний)

      Может случиться так, что уравнение имеет два радикальных выражения.

      Пример 7: Решить: 3x − 4 = 2x + 9.

      Решение: Оба радикала считаются изолированными на разных сторонах уравнения.

      Проверить x = 13.

      Ответ: Решение — 13.

      Пример 8: Решите: x2 + x − 143 = x + 503.

      Решение: Удалите радикалы, срезав кубики с обеих сторон.

      Проверить.

      Ответ: Решения -8 и 8.

      Мы узнаем, как решать некоторые из более сложных радикальных уравнений в следующем курсе, Промежуточная алгебра.

      Попробуй! Решите: 3x + 1 = 2x − 3.

      Ответ: 13

      Основные выводы

      • Решите уравнения, содержащие квадратные корни, сначала выделив радикал, а затем возведя в квадрат обе стороны.Возведение квадратного корня в квадрат устраняет радикал, оставляя нам уравнение, которое можно решить, используя методы, изученные ранее при изучении алгебры. Однако возведение обеих сторон уравнения в квадрат приводит к появлению посторонних решений, поэтому проверьте свои ответы в исходном уравнении.
      • Решите уравнения, содержащие кубические корни, сначала выделив радикал, а затем построив кубы с обеих сторон. Это устраняет радикал и приводит к уравнению, которое можно решить с помощью техник, которые вы уже освоили.

      Тематические упражнения

      Часть A: Решение радикальных уравнений

      Решить.

      1. х = 2

      2. х = 7

      3. х + 7 = 8

      4. х + 4 = 9

      5. х + 6 = 3

      6. х + 2 = 1

      7. 5x − 1 = 0

      8. 3x − 2 = 0

      9. x − 3 = 3

      10.х + 5 = 6

      11. 3x + 1 = 2

      12. 5x − 4 = 4

      13. 7x + 4 + 6 = 11

      14. 3x − 5 + 9 = 14

      15. 2x − 1−3 = 0

      16. 3x + 1-2 = 0

      17. x3 = 2

      18. x3 = 5

      19. 2x + 93 = 3

      20. 4x − 113 = 1

      21. 5x + 73 + 3 = 1

      22. 3x − 63 + 5 = 2

      23. 2 x + 23−1 = 0

      24.2 2х − 33−1 = 0

      25. 8x + 11 = 3x + 1

      26. 23x − 4 = 2 (3x + 1)

      27. 2 (x + 10) = 7x − 15

      28,5 (х − 4) = х + 4

      29. 5x − 23 = 4×3

      30. 9 (x − 1) 3 = 3 (x + 7) 3

      31. 3x + 13 = 2 (x − 1) 3

      32. 9×3 = 3 (x − 6) 3

      33. 4x + 21 = x

      34. 8x + 9 = x

      35,4 (2x − 3) = x

      36,3 (4x − 9) = x

      37.2х − 1 = х

      38. 32x − 9 = x

      39. 9x + 9 = x + 1

      40. 3x + 10 = x + 4

      41. х − 1 = х − 3

      42. 2x − 5 = x − 4

      43. 16−3x = x − 6

      44. 7−3x = x − 3

      45. 32x + 10 = x + 9

      46. 22x + 5 = x + 4

      47. 3x − 1−1 = x

      48. 22x + 2−1 = x

      49. 10x + 41−5 = x

      50.6 (х + 3) −3 = х

      51. 8×2−4x + 1 = 2x

      52. 18×2−6x + 1 = 3x

      53. 5х + 2 = х + 8

      54. 42 (х + 1) = х + 7

      55. x2−25 = x

      56. х2 + 9 = х

      57. 3 + 6x − 11 = x

      58. 2 + 9x − 8 = x

      59. 4x + 25 − x = 7

      60. 8x + 73 − x = 10

      61. 24x + 3−3 = 2x

      62. 26x + 3−3 = 3x

      63.2x − 4 = 14−10x

      64. 3x − 6 = 33−24x

      65. x2−243 = 1

      66. x2−543 = 3

      67. x2 + 6×3 + 1 = 4

      68. x2 + 2×3 + 5 = 7

      69. 25×2−10x − 73 = −2

      70. 9×2−12x − 233 = −3

      71. 2×2−15x + 25 = (x + 5) (x − 5)

      72. x2−4x + 4 = x (5 − x)

      73. 2 (x2 + 3x − 20) 3 = (x + 3) 23

      74. 3×2 + 3x + 403 = (x − 5) 23

      75.х1 / 2−10 = 0

      76. x1 / 2−6 = 0

      77. x1 / 3 + 2 = 0

      78. x1 / 3 + 4 = 0

      79. (x − 1) 1 / 2−3 = 0

      80. (x + 2) 1 / 2−6 = 0

      81. (2x − 1) 1/3 + 3 = 0

      82. (3x − 1) 1 / 3−2 = 0

      83. (4x + 15) 1 / 2−2x = 0

      84. (3x + 2) 1 / 2−3x = 0

      85. (2x + 12) 1/2 − x = 6

      86. (4x + 36) 1/2 − x = 9

      87.2 (5x + 26) 1/2 = x + 10

      88,3 (x − 1) 1/2 = x + 1

      89. Квадратный корень из 1 меньше двойного числа равен 2 меньше числа. Найдите номер.

      90. Квадратный корень из 4 меньше двойного числа равен 6 меньше числа. Найдите номер.

      91. Квадратный корень двойного числа равен половине этого числа. Найдите номер.

      92. Квадратный корень двойного числа равен одной трети этого числа.Найдите номер.

      93. Расстояние, d , измеренное в милях, до которого человек может увидеть объект, определяется по формуле

      д = 6х3

      , где h — высота человека над уровнем моря, измеренная в футах. Насколько высоко должен быть человек, чтобы увидеть объект на расстоянии 5 миль?

      94. Ток I , измеренный в амперах, определяется по формуле

      I = PR

      , где P, — потребляемая мощность, измеренная в ваттах, а R — сопротивление, измеренное в омах.Если лампочка требует 1/2 ампера тока и потребляет 60 Вт мощности, то каково сопротивление лампы?

      Период T маятника в секундах определяется формулой

      Т = 2πL32

      , где L — длина в футах. Для каждой задачи ниже рассчитайте длину маятника с учетом периода. Укажите точное значение и округлите приблизительное значение до ближайшей десятой доли фута.

      95. 1 секунда

      96,2 секунды

      97. 1/2 секунды

      98. 1/3 секунды

      Время, t , в секундах, в течение которого объект находится в свободном падении, определяется формулой

      т = s4

      , где s представляет собой расстояние в футах, на которое упал объект. Для каждой задачи ниже рассчитайте расстояние, на которое объект упадет, учитывая количество времени.

      99. 1 секунда

      100. 2 секунды

      101. 1/2 секунды

      102. 1/4 секунды

      Интерпретации x для любого графика имеют вид ( x , 0), где x — действительное число. Следовательно, чтобы найти x -перехватов, установите y = 0 и решите относительно x .Найдите x -перехватывание для каждого из следующих значений.

      103. y = x − 3−1

      104. y = x + 2−3

      105. у = х − 13 + 2

      106. y = x + 13−3

      Часть B: Обсуждение

      107. Обсудите причины, по которым мы иногда получаем посторонние решения при решении радикальных уравнений. Есть ли когда-нибудь условия, при которых нам не нужно проверять наличие посторонних решений? Почему?

      ответов

      1: 4

      3: 1

      5: Ø

      7: 1/25

      9: 12

      11: 1

      13: 3

      15: 13/4

      17: 8

      19: 9

      21: −3

      23: −15/8

      25: 2

      27: 7

      29: 2

      31: −3

      33: 7

      35: 2, 6

      37: 2

      39: -1, 8

      41: 5

      43: Ø

      45: −3, 3

      47: 2, 5

      49: −4, −4

      51: 1/2

      53: 2, 7

      55: Ø

      57: 10

      59: −6, −4

      61: -1/2, 3/2

      63: Ø

      65: −5, 5

      67: −9, 3

      69: 1/5

      71: 5, 10

      73: −7, 7

      75: 100

      77: −8

      79: 10

      81: −13

      83: 5/2

      85: −6, −4

      87: -2, 2

      89: 5

      91: 8

      93: 1623 футов

      95: 8 / π2≈0.8 футов

      97: 2 / π2≈0,2 фута

      99: 16 футов

      101: 4 фута

      103: (4, 0)

      105: (−7, 0)

      Упрощение радикальных выражений

      Прежде чем вы сможете упростить радикальное выражение, вы должны знать важные

      свойства радикалов

      .

      ПРОДУКТ СОБСТВЕННОСТИ КВАДРАТНЫХ КОРНЕЙ

      Для всех реальных чисел

      а

      а также

      б

      ,

      а

      б

      знак равно

      а

      б

      То есть квадратный корень из произведения совпадает с произведением квадратных корней.

      Есть аналогичное свойство частного:

      Для всех реальных чисел

      а

      а также

      б

      ,

      б

      0

      :

      а

      б

      знак равно

      а

      б

      Поскольку отрицательное число, умноженное на отрицательное, всегда является положительным числом, при извлечении квадратного корня необходимо помнить, что ответ будет как положительным, так и отрицательным числом или выражением. Например

      а

      а

      знак равно

      а

      2

      , а также

      (

      а

      )

      (

      а

      )

      знак равно

      а

      2

      .Такие двойственные ответы мы обычно будем обозначать как

      ±

      а

      .

      УПРОЩЕНИЕ РАДИКАЛА

      Идея здесь состоит в том, чтобы найти идеальный квадратный множитель подкоренного выражения, записать подкоренное выражение как произведение, а затем использовать свойство произведения для упрощения.


      Пример 1:

      Упрощать.

      45

      9

      идеальный квадрат, который также является фактором

      45

      .

      45

      знак равно

      9

      5

      Используйте свойство продукта.

      9

      5

      знак равно

      9

      5

      знак равно

      ±

      3

      5

      Если число под корнем не имеет точных квадратных множителей, то его нельзя упростить.Например, число

      17

      нельзя упростить дальше, потому что единственные факторы

      17

      или же

      17

      а также

      1

      . Итак, нет никаких точных квадратных факторов, кроме

      1

      .


      Пример 2:

      Упрощать.

      12

      3

      Используйте свойство частного, чтобы писать под одним знаком квадратного корня.

      12

      3

      знак равно

      12

      3

      Делить.

      знак равно

      4

      знак равно

      ±

      2

      Выражение считается упрощенным, только если в знаменателе отсутствует знак корня. Если у нас есть радикальный признак, мы должны

      рационализировать знаменатель

      . Это достигается умножением числителя и знаменателя на радикал в знаменателе. Обратите внимание, что здесь мы просто умножаем на специальную форму

      1

      , поэтому значение выражения не меняется.


      Пример 3:

      Упрощать.

      5

      6

      5

      6

      знак равно

      5

      6

      6

      6

      Упрощать.

      знак равно

      30

      6

      Иногда нам нужно использовать комбинацию шагов.


      Пример 4:

      Упрощать.21 год

      9

      21 год

      а также

      9

      имеют общий фактор

      3

      , поэтому уменьшите дробь под радикалом.

      21 год

      9

      знак равно

      7

      3

      знак равно

      7

      3

      Теперь рационализируйте знаменатель.

      7

      3

      3

      3

      знак равно

      21 год

      3

      Мы можем добавить или вычесть два радикальных выражения только в том случае, если подкоренные выражения совпадают.Например,

      17

      +

      13

      не может быть далее упрощен. Но мы можем упростить

      5

      2

      +

      3

      2

      используя

      распределительное свойство

      , потому что подкоренные выражения такие же.

      5

      2

      +

      3

      2

      знак равно

      (

      5

      +

      3

      )

      2

      знак равно

      8

      2

      Будь осторожен! Иногда подкоренные выражения выглядят по-разному, но можно упростить и получить то же подкоренное выражение.


      Пример 5:

      Упрощать.

      50

      +

      32

      Упростите оба радикала:

      50

      +

      32

      знак равно

      25

      2

      +

      16

      2

      знак равно

      ±

      5

      2

      ±

      4

      2

      Теперь подкоренные выражения такие же.

    Добавить комментарий

    Ваш адрес email не будет опубликован. Обязательные поля помечены *